You are on page 1of 64

PART 3D B. Working capital.

BEHAVIOURAL ISSUES C. Total assets available.


194 QUESTIONS D. Shareholders' equity.

[5] Source: CMA 1291 3-9


[1] Source: Publisher
A segment of an organization is referred to
In a responsibility accounting system, the
as an investment center if it has
process in which a supervisor and a
subordinate jointly determine the
A. Authority to make decisions affecting
subordinate's goals and plans for achieving
the major determinants of profit
these goals is
including the power to choose its
markets and sources of supply.
A. Top-down budgeting.
B. Authority to make decisions affecting
B. Bottom-up budgeting.
the major determinants of profit
including the power to choose its
C. Management by objectives.
markets and sources of supply and
significant control over the amount of
D. Management by exception.
invested capital.

C. Authority to make decisions over the


[2] Source: CMA 1292 3-30
most significant costs of operations
Richmond Enterprises is reviewing its
including the power to choose the
policies and procedures in an effort to
sources of supply.
enhance goal congruence throughout the
organization. The processes that are most
D. Authority to provide specialized
likely to encourage this behavior are
support to other units within the
organization.
A. Participatory budgeting, reciprocal
cost allocation, and
management-by-objective performance
[6] Source: CMA 1292 3-24
evaluation.
Managerial performance can be measured in
many different ways, including return on
B. Reciprocal cost allocation,
investment (ROI) and residual income (RI).
zero-base budgeting, and standard
A good reason for using RI instead of ROI is
costing.
that
C. Cost-based transfer pricing, imposed
A. RI can be computed without regard
budgeting, and activity-based costing.
to identifying an investment base.
D. Cost-based transfer pricing,
B. Goal congruence is more likely to be
management-by-objective performance
promoted by using RI.
evaluation, and participatory budgeting.
C. RI is well understood and often used
in the financial press.
[3] Source: CMA 0691 3-24
Most firms use return on investment (ROI)
D. ROI does not take into consideration
to evaluate the performance of investment
both the investment turnover ratio and
center managers. If top management wishes
return-on-sales percentage.
division managers to use all assets without
regard to financing, the denominator in the
ROI calculation will be
[7] Source: CMA 0694 3-27
The Stonebrook Company uses a
A. Total assets available.
performance reporting system that reflects
the company's decentralization of decision
B. Total assets employed.
making. The departmental performance
reports show actual costs incurred during
C. Working capital plus other assets.
the period against budgeted costs. Any
variances from the budget are assigned to
D. Shareholders' equity.
the individual department manager who
controls the costs. Stonebrook is using a
type of system called
[4] Source: CMA 0691 3-29
The selection of the denominator in the
A. Transfer-pricing accounting.
return on investment (ROI) formula is
critical to the measure's effectiveness.
B. Flexible budgeting.
Which denominator is criticized because it
combines the effects of operating decisions
C. Responsibility accounting.
made at one level of the organization with
financing decisions made at another
D. Activity-based budgeting.
organizational level?

A. Total assets employed.


[8] Source: CMA 0694 3-28
DigitalTech uses an accounting system that
charges costs to the manager who has the
authority to make decisions incurring the C. Increasing sales volume while
costs. For example, if a sales manager holding fixed expenses constant.
authorizes a rush order that results in
additional manufacturing costs, these D. None of the answers is correct.
additional costs are charged to the sales
manager. This type of accounting system is
known as [13] Source: CMA 0684 4-9
Return on investment (ROI) is a term often
A. Responsibility accounting. used to express income earned on capital
invested in a business unit. A company's
B. Functional accounting. ROI will increase if

C. Transfer-pricing accounting. A. Sales increase by the same dollar


amount as expenses and total assets
increase.
D. Contribution accounting.
B. Sales remain the same and expenses
are reduced by the same dollar amount
[9] Source: CMA 1294 3-22 that total assets increase.
If a manufacturing company uses
responsibility accounting, which one of the C. Sales decrease by the same dollar
following items is least likely to appear in a amount that expenses increase.
performance report for a manager of an
assembly line? D. Sales and expenses increase by the
same percentage that total assets
increase.
A. Supervisory salaries.

B. Materials. [14] Source: Publisher


To properly motivate divisional
C. Repairs and maintenance. management, the divisional ROIs should be

D. Equipment depreciation. A. Equal.

B. Greater in the less profitable


[10] Source: Publisher divisions to motivate those divisions to
In evaluating an investment center, top achieve higher ROIs.
management should concentrate on
C. Lower in more profitable divisions
A. Dollar sales. in which motivation is unnecessary.

B. Net income. D. Different based upon strategic goals


of the firm.
C. Profit percentages.

D. Return on investment. [15] Source: Publisher


Goal congruence is

[11] Source: Publisher A. The desire and the commitment to


The return on investment calculation achieve a specific goal.
considers only the following components:
B. The sharing of goals by supervisors
S = Sales and subordinates.
I = Investment
NI = Net income C. The extent to which individuals have
Which of the following formulas best the authority to make decisions.
describes the return on investment
calculation? D. The extent of the attempt to
accomplish a specific goal.
A. (I ÷ S) x (S ÷ NI) = I ÷ NI

B. (S ÷ I) x (NI ÷ S) = NI ÷ I [16] Source: Publisher


The proposed transfer price is based upon
C. (I ÷ S) x (NI ÷ S) = (I x NI) x (S x S) the outlay cost. Outlay cost plus opportunity
cost is
D. (S ÷ I) x (S ÷ NI) = (S x S) ÷ (I x NI)
A. The retail price.

[12] Source: Publisher B. The price representing the cash


Which of the following will not improve outflows of the supplying division plus
return on investment if other factors are the contribution to the supplying
constant? division from an outside sale.

A. Decreasing expenses or assets. C. The price usually set by an


absorption-costing calculation.
B. Increasing selling prices.
D. The price set by charging for producing units for B. Division A cannot
variable costs plus a lump sum or an increase its sales to outsiders. From the
additional markup, but less than full perspective of the company as a whole,
markup. from whom should Division B acquire the
units, assuming B's market is unaffected?

[17] Source: Publisher A. Outside vendors.


The proposed transfer price is a cost-plus
price. Variable-cost-plus price is B. Division A, but only at the variable
cost per unit.
A. The price on the open market.
C. Division A, but only until fixed costs
B. The price representing the cash are covered, then from outside vendors.
outflows of the supplying division plus
the contribution to the supplying D. Division A, despite the increased
division from an outside sale. transfer price.

C. The price set by charging for


variable costs plus a lump sum or an [21] Source: CIA 0592 IV-19
additional markup, but less than full Division Z of a company produces a
markup. component that it currently sells to outside
customers for $20 per unit. At its current
D. The price otherwise paid by an level of production, which is 60% of
outsider, recorded by the selling capacity, Division Z's fixed cost of
division but not the buying division. producing this component is $5 per unit and
its variable cost is $12 per unit. Division Y
of the same company would like to purchase
[18] Source: Publisher this component from Division Z for $10.
Full-cost price is Division Z has enough excess capacity to
fill Division Y's requirements. The
A. The price on the open market. managers of both divisions are compensated
based upon reported profits. Which of the
B. The price representing the cash following transfer prices will maximize
outflows of the supplying division plus total company profits and be most equitable
the contribution to the supplying to the managers of Division Y and Division
division from an outside sale. Z?

C. The price usually set by an A. $12 per unit.


absorption-costing calculation.
B. $18 per unit.
D. The price set by charging for
variable costs plus a lump sum or an C. $20 per unit.
additional markup, but less than full
markup. D. $22 per unit.

[19] Source: CIA 1190 IV-20 [22] Source: CIA 0589 IV-16
A limitation of transfer prices based on Division A of a company is currently
actual cost is that they operating at 50% capacity. It produces a
single product and sells all its production to
A. Charge inefficiencies to the outside customers for $13 per unit. Variable
department that is transferring the costs are $7 per unit, and fixed costs are $6
goods. per unit at the current production level.
Division B, which currently purchases this
B. Can lead to suboptimal decisions for product from an outside supplier for $12 per
the company as a whole. unit, would like to purchase the product
from Division A. Division A will operate at
C. Must be adjusted by some markup. 80% capacity to meet outside customers'
and Division B's demand. What is the
D. Lack clarity and administrative minimum price that Division A should
convenience. charge Division B for this product?

A. $7.00 per unit.


[20] Source: CIA 1183 IV-5
A company has two divisions, A and B,
each operated as a profit center. A charges B. $9.60 per unit.
B $35 per unit for each unit transferred to B.
Other data follow: C. $12.00 per unit.

A's variable cost per unit $30 D. $13.00 per unit.


A's fixed costs $10,000
A's annual sales to B 5,000 units
A's sales to outsiders 50,000 units [23] Source: CIA 0588 IV-19
A is planning to raise its transfer price to The alpha division of a company, which is
$50 per unit. Division B can purchase units operating at capacity, produces and sells
at $40 each from outsiders, but doing so 1,000 units of a certain electronic
would idle A's facilities now committed to component in a perfectly competitive
market. Revenue and cost data are as storage and handling are $40. The company
follows: president selects a $220 transfer price. This
is an example of
Sales $50,000
Variable costs 34,000 A. Market-based transfer pricing.
Fixed costs 12,000
The minimum transfer price that should be B. Cost-based transfer pricing.
charged to the beta division of the same
company for each component is C. Negotiated transfer pricing.

A. $12 D. Cost plus 20% transfer pricing.

B. $34
[27] Source: CIA 0594 III-40
C. $46 Which of the following is not true about
international transfer prices for a
D. $50 multinational firm?

A. Allows firms to attempt to minimize


[24] Source: CIA 0593 IV-16 worldwide taxes.
Which of the following is the most
significant disadvantage of a cost-based B. Allows the firm to evaluate each
transfer price? division.

A. Requires internally developed C. Provides each division with a


information. profit-making orientation.

B. Imposes market effects on company D. Allows firms to correctly price


operations. products in each country in which it
operates.
C. Requires externally developed
information.
[28] Source: CIA 0594 III-71
D. May not promote long-term Which of the following is not true of
efficiencies. responsibility accounting?

A. Managers should only be held


[25] Source: CIA 1193 IV-18 accountable for factors over which they
One department of an organization, Final have significant influence.
Assembly, is purchasing subcomponents
from another department, Materials B. The focus of cost center managers
Fabrication. The price that will be charged will normally be more narrow than that
to Final Assembly by Materials Fabrication of profit center managers.
is to be determined. Outside market prices
for the subcomponents are available. Which C. Every factor that affects a firm's
of the following is the most correct financial performance ultimately is
statement regarding a market-based transfer controllable by someone, even if that
price? someone is the person at the top of the
firm.
A. Marginal production cost transfer
prices provide incentives to use D. When a responsibility account
otherwise idle capacity. system exists, operations of the business
are organized into separate areas
B. Market transfer prices provide an controlled by individual managers.
incentive to use otherwise idle
capacity.
[29] Source: CIA 1191 IV-18
C. Overall long term competitiveness is A company plans to implement a bonus plan
enhanced with a market-based transfer based on segment performance. In addition,
price. the company plans to convert to a
responsibility accounting system for segment
D. Corporate politics is more of a reporting. The following costs, which have
factor in a market-based transfer price been included in the segment performance
than with other methods. reports that have been prepared under the
current system, are being reviewed to
determine if they should be included in the
[26] Source: CIA 1193 IV-19 responsibility accounting segment reports:
The Eastern division sells goods internally
to the Western division of the same I. Corporate administrative costs allocated on the basis of
company. The quoted external price in net
industry publications from a supplier near segment sales.
Eastern is $200 per ton plus transportation. II. Personnel costs assigned on the basis of the number of
It costs $20 per ton to transport the goods to employees
Western. Eastern's actual market cost per in each segment.
ton to buy the direct materials to make the III. Fixed computer facility costs divided equally among each
transferred product is $100. Actual per ton segment.
direct labor is $50. Other actual costs of
IV. Variable computer operational costs charged to each
segment based A. Generally accepted accounting
on actual hours used times a predetermined standard principles.
rate; any
variable cost efficiency or inefficiency remains in the B. The Financial Accounting Standards
computer Board.
department.
Of these four cost items, the only item that C. The American Institute of Certified
could logically be included in the segment Public Accountants.
performance reports prepared on a
responsibility accounting basis would be the D. Management.

A. Corporate administrative costs.


[34] Source: CIA 1191 IV-17
B. Personnel costs. The receipt of raw materials used in the
manufacture of products and the shipping of
C. Fixed computer facility costs. finished goods to customers is under the
control of the warehouse supervisor. The
D. Variable computer operational costs. warehouse supervisor's time is spent
approximately 60% on receiving activities
and 40% on shipping activities. Separate
[30] Source: CIA 1192 IV-22 staffs are employed for the receiving and
An organization employs a system of shipping operations. The labor-related costs
internal reporting that furnishes for the warehousing function are as follows:
departmental managers with revenue and
cost information on only those items that are Warehouse supervisor's salary $ 40,000
subject to their control. Items not subject to Receiving clerks' wages 75,000
the manager's control are not included in the Shipping clerks' wages 55,000
performance reports. This method of Employee benefit costs (30% of wage
accounting is known as and salary costs) 51,000
????????
A. Contribution margin reporting. $221,000
========
B. Segment reporting. The company employs a responsibility
accounting system for performance reporting
C. Absorption cost accounting. purposes. The costs are classified on the
report as period or product costs. The total
D. Responsibility accounting. labor-related costs that would be listed on
the responsibility accounting performance
report as product costs under the control of
[31] Source: CIA 0589 IV-15 the warehouse supervisor for the
In a responsibility accounting system, warehousing function would be
managers are accountable for
A. $97,500
A. Variable costs but not for fixed
costs. B. $128,700

B. Product costs but not for period C. $130,000


costs.
D. $169,000
C. Incremental costs.

D. Costs over which they have [35] Source: CIA 1190 IV-21
significant influence. When comparing the residual income of
several investment centers, the validity of
comparisons may be destroyed by
[32] Source: Publisher
In a responsibility accounting system, a A. Peculiarities of each investment
feedback report that focuses on the center.
difference between budgeted amounts and
actual amounts is an example of B. Consistent use of an imputed interest
rate.
A. Management by exception.
C. Common amounts of invested capital
B. Assessing blame. for each investment center.

C. Granting rewards to successful D. None of the answers is correct.


managers.

D. Ignoring other variables for which [36] Source: Publisher


the budgeted goals were met. Transfer pricing should encourage goal
congruence and managerial effort. In a
decentralized organization, it should also
[33] Source: Publisher
The format for internal reports in a encourage autonomous decision making.
responsibility accounting system is Managerial effort is
prescribed by
A. The desire and the commitment to
achieve a specific goal. C. 4

B. The extent to which individuals have D. 20


the authority to make decisions.

C. The extent of the attempt to [41] Source: Publisher


accomplish a specific goal. (Refers to Fact Pattern #1)
For Segment A, ROI is
D. The sharing of goals between units
of an organization. A. 25%

B. 20.8%
[Fact Pattern #1]
C. 33%
Segment A Segment B Segment C
Segment D D. 8.3%
????????? ????????? ????????? ??????
????
Net income $5,000 -- -- $90,000 [42] Source: Publisher
Sales 60,000 $750,000 $135,000 1,800,000 (Refers to Fact Pattern #1)
Investment 24,000 500,000 45,000 -- For Segment B, ROI is
Net income as % of sales -- -- -- --
Turnover of investment -- -- -- -- A. 6%
ROI -- -- 20% 7.5%
Minimum ROI--dollars -- -- -- $120,000 B. 25%
Minimum ROI--% 20% 6% -- --
Residual income -- -0- $2,250 -- C. 20%

[37] Source: Publisher


(Refers to Fact Pattern #1) D. 7.5%
For Segment B, net income as a percentage
of sales is
[43] Source: Publisher
A. 8% (Refers to Fact Pattern #1)
For Segment A, the minimum dollar ROI is
B. 6.67%
A. $30,000
C. 4%
B. $4,800
D. 10%
C. $120,000

[38] Source: Publisher D. $12,000


(Refers to Fact Pattern #1)
For Segment C, net income as a percentage
of sales is [44] Source: Publisher
(Refers to Fact Pattern #1)
A. 5% For Segment B, the minimum dollar ROI is

B. 6.67% A. $30,000

C. 4% B. $6,750

D. 20% C. $4,800

D. $45,000
[39] Source: Publisher
(Refers to Fact Pattern #1)
For Segment C, the turnover of investment is [45] Source: Publisher
(Refers to Fact Pattern #1)
A. 3 For Segment C, the minimum dollar ROI is

B. 1.5 A. $6,750

C. 2.5 B. $4,800

D. 4 C. $120,000

D. $9,000
[40] Source: Publisher
(Refers to Fact Pattern #1)
For Segment D, the turnover of investment is [46] Source: Publisher
(Refers to Fact Pattern #1)
A. 1.5 Assume that the minimum dollar ROI is
$6,750. In Segment C, the minimum
B. 2.5 percentage of ROI is
A. 20% Cost Investment Profit
Centers Centers Centers
B. 6% ??????? ?????????? ???????
A.
C. 15%
Yes Yes Yes
D. 10% B.

Yes No No
[47] Source: Publisher C.
(Refers to Fact Pattern #1)
In Segment D, the minimum percentage of No Yes Yes
ROI is D.

A. 20% No No No

B. 6%
[52] Source: Publisher
C. 15% Periodic internal reports used for
performance evaluation purposes and based
D. 10% on a responsibility accounting system should
not include

[48] Source: Publisher A. Allocated fixed overhead.


(Refers to Fact Pattern #1)
In Segment A, the residual income is B. A distinction between controllable
and noncontrollable costs.
A. $200
C. An organization chart.
B. $12,000
D. Variances between actual and
C. $(30,000) budgeted controllable costs.

D. $(60,000)
[53] Source: Publisher
Responsibility centers accumulate costs for
[49] Source: Publisher control purposes when using
(Refers to Fact Pattern #1)
In Segment D, the residual income is Process Job-Order Activity-
Costing Costing Based Costing
A. $12,000. ??????? ????????? ?????????????
A.
B. $(30,000)
Yes Yes Yes
C. $(60,000) B.

D. $9,000 Yes No Yes


C.

[50] Source: Publisher No Yes No


Production levels are expected to increase D.
within a relevant range. A flexible budget is
used. What are the anticipated effects on the No No No
following?

Fixed Costs Variable Costs [54] Source: Publisher


per Unit per Unit Harris Co.'s income statement for profit
??????????? ?????????????? center No. 12 for August includes
A.
Contribution margin $84,000
Increase Increase Period expenses:
B. Manager's salary $24,000
Depreciation on
Increase No change accommodations 9,600
C. Allocated corporate expenses 6,000 (39,600)
??????? ????????
Decrease Decrease Profit center income $44,400
D. ========
The profit center's manager is most likely
Decrease No change able to control which of the following?

A. $84,000
[51] Source: Publisher
Which of the following types of B. $68,400
responsibility centers include controllable
revenues in their performance reports? C. $60,000
Direct labor 180,000
D. $44,400 Factory overhead 84,000 $ 50,400
General, selling, and
administrative 36,000 57,600
[55] Source: Publisher ???????? ????????
Suboptimization occurs when a management Total $540,000 $108,000
decision benefits the ======== ========
During year 2, Ash produced 360,000
Company Profit Investment hammers, which were sold for $2.40 each.
As a Whole Center Center Georgia's investment in Ash was $600,000
?????????? ?????? ?????????? and $840,000 at January 1, year 1 and
A. January 1, year 2, respectively. Georgia
normally imputes interest on investments at
Yes Yes Yes 18% of average invested capital.
B.
[58] Source: Publisher
Yes No No (Refers to Fact Pattern #2)
C. For the year ended December 31, year 2,
Ash's return on average investment was
No No No
D. A. 18%

No Yes Yes B. 25.7%

C. 30%
[56] Source: Publisher
The following information pertains to D. 36%
Andrew Co. for the year ended December
31:
[59] Source: Publisher
Sales $720,000 (Refers to Fact Pattern #2)
Net income 120,000 Assume that Ash's net operating income was
Capital investment 480,000 $72,000 and its average invested capital
To determine Andrew's return on was $720,000. Determine Ash's residual
investment, which of the following income (loss) for the year ended December
equations should be used? 31, year 2.

A. (720,000/480,000) x A. $(57,600)
(720,000/120,000) = ROI
B. $(79,200)
B. (480,000/720,000) x
(720,000/120,000) = ROI C. $72,000

C. (720,000/480,000) x D. $(36,000)
(120,000/720,000) = ROI

D. (480,000/720,000) x [60] Source: Publisher


(120,000/720,000) = ROI (Refers to Fact Pattern #2)
How many hammers did Ash have to sell in
year 2 to break even?
[57] Source: Publisher
The following information relates to Cinder A. 120,000
Co.'s Northeast Division:
B. 110,000
Sales $600,000
Variable costs 360,000 C. 100,000
Traceable fixed costs 60,000
Average invested capital 120,000 D. 90,000
Imputed interest rate 8%
Cinder's residual income was
[61] Source: Publisher
A. $170,400 (Refers to Fact Pattern #2)
Ash's contribution margin for the year ended
B. $180,000 December 31, year 2 was

C. $189,600 A. $350,000

D. $230,400 B. $324,000

C. $309,600
[Fact Pattern #2]
Georgia Co.'s tool division, Ash, incurred D. $216,000
the following costs for year 2:

Variable Fixed [62] Source: Publisher


???????? ???????? (Refers to Fact Pattern #2)
Direct materials $240,000 Based on Ash's year 2 financial information,
and a projected year 3 production level of
400,000 hammers, what will Ash's
estimated year 3 total costs and expenses be, [66] Source: CMA 1293 3-17
assuming variable costs remain at $1.50 per The budgeting process that uses management
unit? by objectives and input from the individual
manager is an example of the application of
A. $648,000
A. Flexible budgeting.
B. $708,000
B. Human resource management.
C. $718,800
C. Responsibility accounting.
D. $720,000
D. Capital budgeting.

[63] Source: CIA 0594 III-44


A firm prepared a segmented income [67] Source: CMA 1293 3-21
statement that included the following data A successful responsibility accounting
for its suburban marketing segment: reporting system is dependent upon

Fixed costs controllable by the suburban A. The correct allocation of


marketing segment manager $150,000 controllable variable costs.
Fixed suburban marketing costs
controllable by corporate management $250,000 B. Identification of the management
Fixed manufacturing costs allocated to level at which all costs are
the suburban marketing segment $110,000 controllable.
Variable manufacturing costs $200,000
Variable selling costs $100,000 C. The proper delegation of
Variable administrative costs $130,000 responsibility and authority.
Net sales $950,000
The best measure of the economic D. A reasonable separation of costs into
performance of the suburban marketing their fixed and variable components
segment is: since fixed costs are not controllable
and must be eliminated from the
A. $370,000 responsibility report.

B. $10,000
[68] Source: CMA 0694 3-24
C. $520,000 Decentralized firms can delegate authority
and yet retain control and monitor managers'
D. $120,000 performance by structuring the organization
into responsibility centers. Which one of the
following organizational segments is most
[64] Source: CMA 1291 3-10 like an independent business?
A segment of an organization is referred to
as a service center if it has A. Revenue center.

A. Responsibility for developing B. Profit center.


markets and selling the output of the
organization. C. Cost center.

B. Responsibility for combining the raw D. Investment center.


materials, direct labor, and other
factors of production into a final output.
[69] Source: CMA 1294 3-20
C. Authority to make decisions affecting Fairmount Inc. uses an accounting system
the major determinants of profit that charges costs to the manager who has
including the power to choose its been delegated the authority to make the
markets and sources of supply. decisions incurring the costs. For example,
if the sales manager accepts a rush order
D. Authority to provide specialized that will result in higher than normal
support to other units within the manufacturing costs, these additional costs
organization. are charged to the sales manager because the
authority to accept or decline the rush order
was given to the sales manager. This type of
[65] Source: CMA 0693 3-14 accounting system is known as
The least complex segment or area of
responsibility for which costs are allocated A. Responsibility accounting.
is a(n)
B. Functional accounting.
A. Profit center.
C. Reciprocal allocation.
B. Investment center.
D. Transfer price accounting.
C. Contribution center.

D. Cost center. [70] Source: CMA 1294 3-21


Sherman Company uses a performance line of data for each subordinate who
reporting system that reflects the company's reports to the group vice president. The data
decentralization of decision making. The presented show the actual costs incurred
departmental performance report shows one during the period, the budgeted costs, and
line of data for each subordinate who all variances from budget for that
reports to the group vice president. The data subordinate's department. The WK Company
presented show the actual costs incurred is using a system called
during the period, the budgeted costs, and
all variances from budget for that A. Flexible budgeting.
subordinate's department. Sherman is using
a type of system called B. Responsibility accounting.

A. Contribution accounting. C. Cost-benefit accounting.

B. Cost-benefit accounting. D. Program budgeting.

C. Flexible budgeting.
[75] Source: CMA 0691 3-28
D. Responsibility accounting. The basic purpose of a responsibility
accounting system is

[71] Source: CMA 0695 3-28 A. Budgeting.


In responsibility accounting, a center's
performance is measured by controllable B. Motivation.
costs. Controllable costs are best described
as including C. Authority.

A. Direct material and direct labor, D. Variance analysis.


only.

B. Only those costs that the manager can [76] Source: CMA 0693 3-29
Which one of the following firms is likely to
influence in the current time period. experience dysfunctional motivation on the
part of its managers due to its allocation
C. Only discretionary costs. methods?

D. Those costs about which the manager A. To allocate depreciation of forklifts


is knowledgeable and informed. used by workers at its central
warehouse, Shahlimar Electronics uses
predetermined amounts calculated on
[72] Source: CMA 1295 3-5 the basis of the long-term average use
Responsibility accounting defines an of the services provided.
operating center that is responsible for
revenue and costs as a(n) B. Manhattan Electronics uses the sales
revenue of its various divisions to
A. Profit center. allocate costs connected with the
upkeep of its headquarters building. It
B. Revenue center. also uses ROI to evaluate the divisional
performances.
C. Division.
C. Rainier Industrial does not allow its
D. Operating unit. service departments to pass on their
cost overruns to the production
departments.
[73] Source: CMA 1296 3-16
Rockford Manufacturing Corporation uses a D. Tashkent Auto's MIS is operated out
responsibility accounting system in its of headquarters and serves its various
operations. Which one of the following divisions. Tashkent's allocation of the
items is least likely to appear in a
performance report for a manager of one of MIS-related costs to its divisions is
Rockford's assembly lines? limited to costs the divisions will incur
if they were to outsource their MIS
A. Direct labor. needs.

B. Materials.
[77] Source: CMA 0686 4-14
C. Repairs and maintenance. The segment margin of the Wire Division of
Lerner Corporation should not include
D. Depreciation on the manufacturing
facility. A. Net sales of the Wire Division.

B. Fixed selling expenses of the Wire


[74] Source: CMA 0692 3-23 Division.
The WK Company uses a performance
reporting system that reflects the company's C. Variable selling expenses of the
decentralization of decision making. The Wire Division.
departmental performance report shows one
D. The Wire Division's fair share of the
salary of Lerner Corporation's C. Working capital.
president.
D. Shareholders' equity.

[78] Source: CIA 0587 IV-15


Overtime conditions and pay were recently [82] Source: CMA 1291 3-7
set by the personnel department. The Which one of the following items would
production department has just received a most likely not be incorporated into the
request for a rush order from the sales calculation of a division's investment base
department. The production department when using the residual income approach
protests that additional overtime costs will for performance measurement and
be incurred as a result of the order. The evaluation?
sales department argues that the order is
from an important customer. The production A. Fixed assets employed in division
department processes the order. To control operations.
costs, which department should never be
charged with the overtime costs generated B. Land being held by the division as a
as a result of the rush order? site for a new plant.

A. Personnel department. C. Division inventories when division


management exercises control over the
B. Production department. inventory levels.

C. Sales department. D. Division accounts payable when


division management exercises control
D. Shared by production department over the amount of short-term credit
and sales department. used.

[79] Source: CMA 0691 3-26 [83] Source: CMA 1292 3-21
If a manufacturing company uses A firm earning a profit can increase its
responsibility accounting, which one of the return on investment by
following items is least likely to appear in a
performance report for a manager of an A. Increasing sales revenue and
assembly line? operating expenses by the same dollar
amount.
A. Supervisory salaries.
B. Decreasing sales revenues and
B. Materials. operating expenses by the same
percentage.
C. Repairs and maintenance.
C. Increasing investment and operating
D. Depreciation on equipment. expenses by the same dollar amount.

D. Increasing sales revenues and


[80] Source: CMA 1292 3-22 operating expenses by the same
When using a contribution margin format for percentage.
internal reporting purposes, the major
distinction between segment manager
performance and segment performance is [Fact Pattern #3]
Edith Carolina, president of the Deed
A. Unallocated fixed cost. Corporation, requires a minimum return on
investment of 8% for any project to be
B. Direct variable costs of producing undertaken by her company. The company is
the product. decentralized, and leaves investment
decisions up to the discretion of the division
C. Direct fixed cost controllable by the managers as long as the 8% return is
segment manager. expected to be realized. Michael Sanders,
manager of the Cosmetics Division, has had
D. Direct fixed cost controllable by a return on investment of 14% for his
others. division for the past 3 years and expects the
division to have the same return in the
coming year. Sanders has the opportunity to
[81] Source: CMA 0691 3-29 invest in a new line of cosmetics which is
The selection of the denominator in the expected to have a return on investment of
return on investment (ROI) formula is 12%.
critical to the measure's effectiveness.
Which denominator is criticized because it [84] Source: CMA 0693 3-12
combines the effects of operating decisions (Refers to Fact Pattern #3)
made at one level of the organization with If the Deed Corporation evaluates
financing decisions made at another managerial performance using residual
organizational level? income based on the corporate minimum
required rate of return, what will be the
A. Total assets available. preference for taking on the proposed
cosmetics line by Edith Carolina and
B. Total assets employed. Michael Sanders?
B. Historical weighted-average cost of
Carolina Sanders capital for the company.
???????? ???????
A. C. Target return on investment set by the
company's management.
Accept Reject
B. D. Average return on investments for
the company over the last several years.
Reject Accept
C.
[88] Source: CMA 0694 3-29
Accept Accept
D. One approach to measuring divisional
performance is return on investment. Return
Reject Reject on investment is expressed as operating
income

[85] Source: CMA 0693 3-11 A. Divided by the current year's capital
(Refers to Fact Pattern #3) expenditures plus cost of capital.
If the Deed Corporation evaluates
managerial performance using return on B. Minus imputed interest charged for
investment, what will be the preference for invested capital.
taking on the proposed cosmetics line by
Edith Carolina and Michael Sanders? C. Divided by fixed assets.

Carolina Sanders D. Divided by total assets.


???????? ???????
A.
[89] Source: CMA 0695 3-20
Accept Reject REB Service Co. is a computer service
B. center. For the month of May 1995, REB
had the following operating statistics:
Reject Accept
C. Sales $450,000
Operating income 25,000
Accept Accept Net profit after taxes 8,000
D. Total assets 500,000
Shareholders' equity 200,000
Reject Reject Cost of capital 6%
Based on the above information, which one
of the following statements is correct? REB
[86] Source: CMA 0693 3-27 has a
Which one of the following statements
pertaining to the return on investment (ROI) A. Return on investment of 4%.
as a performance measurement is incorrect?
B. Residual income of $(5,000).
A. When the average age of assets
differs substantially across segments of C. Return on investment of 1.6%.
a business, the use of ROI may not be
appropriate. D. Residual income of $(22,000).

B. ROI relies on financial measures that


are capable of being independently [90] Source: CMA 1296 3-2
verified while other forms of The segment margin of an investment center
performance measures are subject to after deducting the imputed interest on the
manipulation. assets used by the investment center is
known as
C. The use of ROI may lead managers
to reject capital investment projects that A. Return on investment.
can be justified by using discounted
cash flow models. B. Residual income.

D. The use of ROI can make it C. Operating income.


undesirable for a skillful manager to
take on trouble-shooting assignments D. Return on assets.
such as those involving turning around
unprofitable divisions.
[91] Source: CMA 1296 3-27
Which denominator used in the return on
[87] Source: CMA 0694 3-18 investment (ROI) formula is criticized
The imputed interest rate used in the because it combines the effects of operating
residual income approach to performance decisions made at one organizational level
evaluation can best be described as the with financing decisions made at another
organizational level?
A. Average lending rate for the year
being evaluated. A. Total assets employed.
B. Shareholders' equity. The market price of the video card used by
the Entertainment Division is $10.98 per
C. Working capital plus other assets. unit.

D. Total assets available. [94] Source: CMA 0696 3-26


(Refers to Fact Pattern #4)
A per-unit transfer price from the Video
[92] Source: CMA 0692 3-14 Cards Division to the Entertainment
The most fundamental responsibility center
affected by the use of market-based transfer Division at full cost, $9.15, would
prices is a(n)
A. Allow evaluation of both divisions
A. Production center. on a competitive basis.

B. Investment center. B. Satisfy the Video Cards Division's


profit desire by allowing recovery of
C. Cost center. opportunity costs.

D. Profit center. C. Provide no profit incentive for the


Video Cards Division to control or
reduce costs.
[93] Source: CMA 0694 3-30
An appropriate transfer price between two D. Encourage the Entertainment
divisions of The Stark Company can be Division to purchase video cards from
determined from the following data: an outside source.

Fabricating Division
Market price of subassembly $50 [95] Source: CMA 0696 3-27
Variable cost of subassembly $20 (Refers to Fact Pattern #4)
Excess capacity (in units) 1,000 Assume that the Entertainment Division is
Assembling Division able to purchase a large quantity of video
Number of units needed 900 cards from an outside source at $8.70 per
What is the natural bargaining range for the unit. The Video Cards Division, having
two divisions? excess capacity, agrees to lower its transfer
price to $8.70 per unit. This action would
A. Between $20 and $50.
A. Optimize the profit goals of the
B. Between $50 and $70. Entertainment Division while
subverting the profit goals of Parkside
C. Any amount less than $50. Inc.

D. $50 is the only acceptable price. B. Allow evaluation of both divisions


on the same basis.

[Fact Pattern #4] C. Subvert the profit goals of the Video


Parkside Inc. has several divisions that Cards Division while optimizing the
operate as decentralized profit centers. profit goals of the Entertainment
Parkside's Entertainment Division Division.
manufactures video arcade equipment using
the products of two of Parkside's other D. Optimize the overall profit goals of
divisions. The Plastics Division Parkside Inc.
manufactures plastic components, one type
that is made exclusively for the
Entertainment Division, while other less [96] Source: CMA 0696 3-28
complex components are sold to outside (Refers to Fact Pattern #4)
markets. The products of the Video Cards Assume that the Plastics Division has excess
Division are sold in a competitive market; capacity and it has negotiated a transfer
however, one video card model is also used price of $5.60 per plastic component with
by the Entertainment Division. The actual the Entertainment Division. This price will
costs per unit used by the Entertainment
Division are presented below. A. Cause the Plastics Division to
reduce the number of commercial
Plastic Video plastic components it manufactures.
Components Cards
?????????? ????? B. Motivate both divisions as estimated
Direct material $1.25 $2.40 profits are shared.
Direct labor 2.35 3.00
Variable overhead 1.00 1.50 C. Encourage the Entertainment
Fixed overhead .40 2.25 Division to seek an outside source for
????? ????? plastic components.
Total cost $5.00 $9.15
===== ===== D. Demotivate the Plastics Division
The Plastics Division sells its commercial causing mediocre performance.
products at full cost plus a 25% markup and
believes the proprietary plastic component
made for the Entertainment Division would
sell for $6.25 per unit on the open market. [97] Source: CMA 1296 3-17
In theory, the optimal method for D. The price set by charging for
establishing a transfer price is variable costs plus a lump sum or an
additional markup, but less than full
A. Flexible budget cost. markup.

B. Incremental cost.
[102] Source: CIA 1191 IV-19
C. Budgeted cost with or without a A carpet manufacturer maintains a retail
markup. division consisting of stores stocking its
brand and other brands, and a manufacturing
D. Market price. division that makes carpets and pads. An
outside market exists for carpet padding
material in which all padding produced can
[98] Source: CIA 1188 IV-23 be sold. The proper transfer price for
The price that one division of a company padding transferred from the manufacturing
charges another division for goods or division to the retail division is
services provided is called the
A. Variable manufacturing division
A. Market price. production cost.

B. Transfer price. B. Variable manufacturing division


production cost plus allocated fixed
C. Outlay price. factory overhead.

D. Distress price. C. Variable manufacturing division


production cost plus variable selling
and administrative cost.
[99] Source: Publisher
Motivation is D. The market price at which the retail
division could purchase padding.
A. The desire and the commitment to
achieve a specific goal.
[103] Source: CIA 1190 IV-20
B. The sharing of goals by supervisors A limitation of transfer prices based on
and subordinates. actual cost is that they

C. The extent to which individuals have A. Charge inefficiencies to the


the authority to make decisions. department that is transferring the
goods.

D. The extent of the attempt to B. Can lead to suboptimal decisions for


accomplish a specific goal. the company as a whole.

C. Must be adjusted by some markup.


[100] Source: Publisher
Outlay cost plus opportunity cost is D. Lack clarity and administrative
convenience.
A. The retail price.

B. The price representing the cash [104] Source: CIA 0595 III-96
outflows of the supplying division plus Which of the following techniques would be
the contribution to the supplying best for evaluating the management
division from an outside sale. performance of a department that is operated
as a cost center?
C. The price usually set by an
absorption-costing calculation. A. Return on assets ratio.

D. The price set by charging for B. Return on investment ratio.


variable costs plus a lump sum or an
additional markup, but less than full C. Payback method.
markup.
D. Variance analysis.

[101] Source: Publisher


Variable-cost-plus price is [105] Source: CMA 0697 3-29
Listed below is selected financial
A. The price on the open market. information for the Western Division of the
Hinzel Company for last year.
B. The price representing the cash
outflows of the supplying division plus Amount
the contribution to the supplying Account (thousands)
??????????????????????????????????? ??????????
division from an outside sale. ?
Average working capital $ 625
C. The price usually set by an General and administrative expenses 75
absorption-costing calculation. Net sales 4,000
Average plant and equipment 1,775
Cost of goods sold 3,525 assembly line?
If Hinzel treats the Western Division as an
investment center for performance A. Supervisory salaries.
measurement purposes, what is the
before-tax return on investment (ROI) for
last year? B. Materials.

A. 34.78% C. Repairs and maintenance.

B. 22.54% D. Depreciation on equipment.

C. 19.79%
[109] Source: Publisher
D. 16.67% Charlie's Service Co. is a service center.
For the month of June, Charlie's had the
following operating statistics:
[106] Source: CMA 0697 3-24
Residual income is a better measure for Sales $750,000
performance evaluation of an investment Operating income 25,000
center manager than return on investment Net profit after taxes 8,000
because Total assets available 500,000
Shareholders' equity 200,000
A. The problems associated with Cost of capital 6%
measuring the asset base are eliminated. Charlie's has a

B. Desirable investment decisions will A. Return on investment of 3.33%.


not be neglected by high-return
divisions. B. Residual income of $(5,000).

C. Only the gross book value of assets C. Return on investment of 6%.


needs to be calculated.
D. Residual income of $(20,000).
D. The arguments about the implicit
cost of interest are eliminated.
[110] Source: CMA Samp Q3-2
Consider the following information for
[107] Source: Publisher Richardson Company for the prior year.
The following forecasted information is
available for a manufacturing division for - The company produced 1,000 units and sold 900 units,
next year: both as
budgeted.
Amount - There were no beginning or ending work-in-process
Category (thousands) inventories and no
??????????????????? ??????????? beginning finished goods inventory.
Working capital $ 1,800 - Budgeted and actual fixed costs were equal, all variable
Revenue 30,000 manufacturing costs were affected by production volume
Plant and equipment 17,200 only, and all
To establish a standard of performance for variable selling costs were affected by sales volume only.
the division's manager using the residual - Budgeted per unit revenues and costs were as follows:
income approach, four scenarios are being Per unit
considered. Scenario 1 assumes an imputed ????????
interest charge of 15% and a target residual Sales price $100
income of $2,000,000. Scenario 2 assumes Direct materials 30
an imputed interest charge of 12% and a Direct labor 20
target residual income of $1,500,000. Other variable manufacturing costs 10
Scenario 3 assumes an imputed interest Fixed selling costs 5
charge of 18% and a target residual income Variable selling costs 12
of $1,250,000. Scenario 4 assumes an Fixed selling costs ($33,600 total) 4
imputed interest charge of 10% and a target Fixed administrative costs ($1,800 total) 2
residual income of $2,500,000. Which of the The contribution margin earned by
scenarios assumes the lowest maximum Richardson for the prior year was
cost?
A. $25,200
A. Scenario 1.
B. $28,000
B. Scenario 2.
C. $31,500
C. Scenario 3.
D. $35,000
D. Scenario 4.

[111] Source: Publisher


[108] Source: CMA 0691 3-26 Assume Avionics Industrials reported at
If a manufacturing company uses year-end that operating income before taxes
responsibility accounting, which one of the for the year equaled $2,400,000. Long-term
following items is least likely to appear in a debt issued by Avionics has a coupon rate
performance report for a manager of an equal to 6%, and its cost of equity is 8%.
The book value of the debt currently equals Revenue 30,000
its fair value, and the book value of the Plant and equipment 17,200
equity capital for Avionics is $900,000 less If the imputed interest charge is 15% and
than its fair value. Current assets are listed Webb wants to achieve a residual income
at $2,000,000 and long-term assets equal target of $2,000,000, what will costs have
$9,600,000. The claims against those assets to be in order to achieve the target?
are in the form of $1,500,000 in current
liabilities and $2,200,000 in long-term A. $9,000,000
liabilities. The income tax rate for Avionics
is 30 percent. What is the economic value B. $10,800,000
added (EVA)?
C. $25,150,000
A. $731,240
D. $25,690,000
B. $948,760

C. $1,668,760 [115] Source: Publisher


An effective managerial control system has
D. $1,680,000 favorable motivational effects. Which of the
following are aspects of motivation?

[112] Source: Publisher Managerial Goal


What is the weighted-average cost of capital Effort Congruence
(WACC) to be used in the economic value ?????????? ??????????
added (EVA) calculation? A.

A. 8% No No
B.
B. 8.89%
No Yes
C. 9% C.

D. 10% Yes Yes


D.

[Fact Pattern #5] Yes No


Dzyubenko Co. reported these data at
year-end:
[116] Source: Publisher
Pre-tax operating income $ 4,000,000 Goal congruence is most likely to be
Current assets 4,000,000 promoted when
Long-term assets 16,000,000
Current liabilities 2,000,000 A. A company uses a cost-based
Long-term liabilities 5,000,000 transfer price.
The long-term debt has an interest rate of
8%, and its fair value equaled its book B. A manager of a retail store is
value at year-end. The fair value of the charged imputed interest on inventory.
equity capital is $2 million greater than its
book value. Dzyubenko's income tax rate is C. Percentage return on investment
25%, and its cost of equity capital is 10%. rather than residual income is used to
measure managerial performance.
[113] Source: Publisher
(Refers to Fact Pattern #5) D. An annual accrual accounting
The EVA is measure is used to evaluate a manager
who makes capital investment
A. $1,380,000 decisions.

B. $1,620,000
[117] Source: CIA 1195 II-36
C. $1,830,000 To make goal setting effective and
worthwhile, the goals should be
D. $3,000,000
A. Just beyond what subordinates are
likely to reach.
[114] Source: CMA 0697 3-30
(Refers to Fact Pattern #5) B. Qualitative and approximate.
James Webb is the general manager of the
Industrial Product Division, and his C. Based on superior performers'
performance is measured using the residual output.
income method. Webb is reviewing the
following forecasted information for his D. Specific, objective, and verifiable.
division for next year:

Amount [118] Source: Publisher


Category (thousands) The extent to which a manager can influence
??????????????????? ??????????? organizational activities is
Working capital $ 1,800
A. Authority. [122] Source: Publisher
MBO managers are most likely to believe
B. Responsibility. that employees

C. Accountability. A. Dislike their work.

D. Controllability. B. Avoid responsibility whenever


possible.

[119] Source: CMA 0693 3-21 C. Work best when threatened with
Which one of the following organizational punishment.
policies is most likely to result in
undesirable managerial behavior? D. Are self-motivated.

A. Patel Chemicals sponsors television


coverage of cricket matches between [123] Source: Publisher
national teams representing India and A budget is often the result of a
Pakistan. The expenses of such media management-by-objectives (MBO) program.
sponsorship are not allocated to its A characteristic of MBO is
various divisions.
A. Development of a single measure of
B. Joe Walk, the chief executive officer employee performance.
of Eagle Rock Brewery, wrote a
memorandum to his executives stating, B. Statement of objectives in general
"Operating plans are contracts, and they terms.
should be met without fail."
C. Establishment of objectives through
C. The budgeting process at Madsen both top-down and bottom-up
Manufacturing starts with operating processes.
managers providing goals for their
respective departments. D. A flexible time frame for
achievement of objectives.
D. Fullbright Lighting holds quarterly
meetings of departmental managers to
consider possible changes in the [124] Source: Publisher
budgeted targets due to changing A company has a compensation system for
conditions. its managers based on an MBO approach.
The essential premise of MBO is that

[120] Source: Publisher A. Compensation should be based on


An advantage of participatory budgeting is qualitative factors.
that it
B. Employees should be concerned with
A. Minimizes the cost of developing routine matters, and managers should
budgets. attend to exceptions.

B. Reduces the effect on the budgetary C. Employees should participate in


process of employee biases. setting goals.

C. Yields information known to D. Managers should establish goals for


management but not to employees. their employees.

D. Encourages acceptance of the budget


by employees. [125] Source: Publisher
Which of the following is a weakness of
using MBO for motivating employees?
[121] Source: CMA 0693 3-26
Which one of the following will not occur in A. Employees may believe that they
an organization that gives managers control situations that arise in the
throughout the organization maximum company.
freedom to make decisions?
B. Employees may be forced to place
A. More effective solutions to too much emphasis on quantitative
operational problems. factors.

B. Individual managers regarding the C. Employees may participate too much


managers of other segments as they do in the goal-setting process.
external parties.
D. Employees may become too trusting
C. Two divisions of the organization and too dependent on the "team" to get
having competing models that aim for things done.
the same market segments.

D. Delays in securing approval for the [126] Source: Publisher


introduction of new products. Managerial performance may be measured
in many ways. For example, an internal
nonfinancial measure is
A. Market share. D. Improvement in managerial morale
and the potential for behavioral change.
B. Delivery performance.

C. Customer satisfaction. [130] Source: Publisher


Common costs are
D. Manufacturing lead time.
A. Direct costs.

[127] Source: Publisher B. Current costs.


Many forms of performance feedback are
based on accounting information. For C. Controllable costs.
example, a divisional manager may be
evaluated based on return on investment D. Indirect costs.
(income ÷ investment). One step in the
process of developing a performance
measure based on accounting information is [131] Source: Publisher
to determine the basis for stating the Managers are most likely to accept
measure in terms of dollars. Thus, if ROI is allocations of common costs based on
the chosen measure, and investment is
defined as total assets, comparability issues A. Cause and effect.
are most likely to arise when the attribute
used to calculate total assets is B. Ability to bear.

A. Current cost. C. Fairness.

B. Current disposal price. D. Benefits received.

C. Historical cost.
[132] Source: Publisher
D. Present value. A large corporation allocates the costs of its
headquarters staff to its decentralized
divisions. The best reason for this allocation
[128] Source: Publisher is to
An organization's managerial
decision-making model for capital budgeting A. More accurately measure divisional
is based on the net present value of operating results.
discounted cash flows. The same
organization's managerial performance B. Improve divisional managements'
evaluation model is based on annual morale.
divisional return on investment. Which of
the following is true? C. Remind divisional managers that
common costs exist.
A. Divisional managers are likely to
maximize the measures in the D. Discourage any use of central
decision-making model. support services.

B. Divisional managers are likely to


maximize the measures in the [133] Source: Publisher
performance evaluation model. Which budget is most likely to facilitate
variance analysis?
C. The manager has an incentive to
accept a project with a positive net A. Fixed budget.
present value that initially has a
negative effect on net income. B. Static budget.

D. The use of models with different C. Continuous budget.


criteria promotes goal congruence.
D. Flexible budget.

[129] Source: Publisher


In a well-run organization, a manager may [134] Source: Publisher
have responsibility for activities that (s)he Using the balanced scorecard approach, an
cannot significantly influence. This organization evaluates managerial
arrangement may be justified because of the performance based on

A. Manager's knowledge about the A. A single ultimate measure of


activities and the potential for operating results, such as residual
behavioral change. income.

B. Manager's knowledge about the B. Multiple financial and nonfinancial


activities but not the potential for measures.
behavioral change.
C. Multiple nonfinancial measures only.
C. Improvement in managerial morale,
effort, and performance. D. Multiple financial measures only.
their culpability.

[135] Source: CIA 0594 III-50 D. Focusing the discussion on the


Which of the following is not an advantage offending behavior instead of the
of teamwork compared with work offender.
performed by individuals?

A. Teams provide support to the team [140] Source: CIA 0594 III-83
members. An organization's management perceives the
need to change fundamentally. Which of the
B. Teams make decisions that are more following factors is management least likely
easily accepted. to change?

C. Teams provide a clear link between A. The organization's members.


effort and outcome.
B. The organization's structure.
D. Teams control and discipline
members. C. The organization's environment.

D. The organization's technology.


[136] Source: CIA 1193 III-2
Which of the following is most likely to
provide for the continual development of [141] Source: CIA 1194 II-26
managerial skills? While conducting fieldwork, a strong
conflict arises between two of your
A. Organizational orientation sessions. subordinates regarding possible scope
expansion. You draw their attention to their
B. Job rotation. shared views, downplaying the issues of
contention. This technique for resolving
C. Role playing. conflict is called

D. Liaison committees. A. Superordinate goals.

B. Smoothing.
[137] Source: CIA 0594 III-81
Which of the following would not be C. Problem solving.
considered a conflict trigger?
D. Compromise.
A. Ambiguous jurisdictions.

B. Competition for scarce resources. [142] Source: CIA 0595 II-32


Upon completing an audit of a major
C. Status differential. division of an international chemical
manufacturer, the auditor noted that the
D. Superordinate goals. division seemed to focus almost solely on
the attainment of profits -- with little
attention paid to other issues. The auditor
[138] Source: CIA 0594 II-9 had noted that the company's mission
Which of the following is not an example of statement emphasized generating returns to
positive reinforcement of behavior? the stockholders. The profit emphasis was
consistent with the mission statement. This
A. Paying a bonus to employees who is an example of
had no absences for any four-week
period. I. A goal attainment approach in which the ends are the
focus
B. Giving written warnings after only of attention and the means are evaluated only for their
every other absence. promotion of the ends
II. An approach that is best suited for a company or division
C. Assigning a mentor to each at
employee. the formalization or initialization phase of its business
III. An effective implementation of a strategic constituencies
D. Having a lottery every month where approach because the organization is clearly focused on
10% of the employees with no absences developing returns for its stockholders
receive a $200 bonus. A. I only.

B. I and II only.
[139] Source: CIA 0594 III-89
The process of disciplining employees is C. III only.
made less effective by:
D. I, II, and III.
A. Stating the offending behavior
specifically.
[143] Source: CIA 0595 II-35
B. Postponing the start of disciplinary The manager faces two potentially
procedures. conflicting trends when formulating the
presentation of audit reports: (A) the
C. Permitting employees to challenge information age and the increase in
information available for decision making; based on this observation?
and (B) the concept of "bounded
rationality," which pertains to the ability of A. This supervisor is too lenient in
the individuals to process data and make performance ratings.
decisions. Which of the following
approaches to developing a report would be B. The performance appraisal forms
consistent with the notion of bounded probably have too many leading
rationality? questions.

A. Identify the information most C. There may be a systematic bias in the


available to encourage the user to make way individuals are being rated.
an optimal decision.
D. This supervisor is trying too hard to
B. Identify the problem, but let the treat everyone the same.
decision maker independently search
the databases for the problem solution.
[147] Source: CIA 1195 II-16
C. Present data in graphic form to focus Which one of the following techniques is not
on the major, more simple aspects of generally recognized as an effective conflict
complex problems. resolution technique for management to use
in a dispute between employees?
D. Suggest that a short time limit be
placed on the time to make a decision A. Command - management resolves the
so that the decision maker will conflict by using its formal authority.
concentrate on taking effective action in
a timely fashion. B. Competition - management
encourages the parties to seek their own
interests, regardless of the effect on
[144] Source: CIA 0595 II-34 each other.
There has been an increased emphasis on
group decision making in organizations. C. Reorganization - management
Which of the following statements has been transfers one of the disputants to another
found to hold true in studies of individual department.
decision making as compared to group
decision making? Individual decision D. Compromise - management
making tends to persuades each party to make
concessions.
A. Be more conservative.

B. Evaluate more complete information. [148] Source: CIA 1195 II-18


A manager must design a program to help
C. Generate more alternatives. subordinates learn about a new procedure
that an organization has implemented and is
D. Be more time consuming. considering both classroom lecture and
comprehensive on-the-job (experiential)
training approaches. Which one of the
[145] Source: CIA 1195 II-1 following is a disadvantage of the
Which of the following statements about traditional classroom lecture format as a
group decision making is generally training method when compared to learning
considered false? by experience?

A. There is a lack of responsibility for A. It is more expensive.


group decisions.
B. It takes longer.
B. Group decision making is almost
always less efficient than individual C. It is difficult to go in-depth on a
decision making. given topic.

C. Although group members frequently


have diverse views about a decision, D. Trainees do not easily retain what
each member's desire to be accepted by they learn in the classroom.
the group often restrains open
disagreement.
[149] Source: CIA 1195 II-33
D. Group decision making tends to be Which one of the following statements about
less creative than individual decision quality circles is false?
making.
A. A quality circle is typically
comprised of a group of 8 to 10
[146] Source: CIA 1195 II-15 subordinates and supervisors.
A manager notices that a particular
supervisor's performance appraisal ratings B. Part of the quality circle concept
for any given subordinate tend to be includes teaching participants
consistently high, consistently low, or communication skills, quality strategies,
consistently in the middle across all and problem analysis techniques.
performance dimensions. Which one of the
following is the best conclusion to draw C. Quality circles meet on the company
premises and on company time. of group decision making as compared to
individual decision making?
D. The quality circle has the final
control over implementation of A. Groups obtain an increased degree
recommended solutions. of acceptance of a solution so that it
may be more easily implemented.

[150] Source: CIA 1195 II-36


To make goal-setting effective and B. Group decision making is consistent
worthwhile, the goals should be with democratic methods.

A. Just beyond what subordinates are C. Group members bring more complete
likely to reach. information and knowledge into the
decision process.
B. Qualitative and approximate.
D. Group members avoid expressing
C. Based on superior performers' opinions that deviate from what appears
output. to be the group consensus.

D. Specific, objective, and verifiable.


[155] Source: CIA 0596 II-20
The best way a new employee can learn
[151] Source: CIA 1194 II-28 about the company's corporate culture
The least effective method for an employee would be to
to learn computer skills is
A. Seek the advice of a more
A. Classroom training using equipment. experienced person who was hired at
the same time to learn about how the
B. Video training. company works.

C. Apprenticeships. B. Read professional literature and


journals to ascertain how experts view
D. Case analysis. the company.

C. Watch the behavior of others to


[152] Source: CIA 1196 II-38 determine what works and what does
Which of the following statements is correct not.
with respect to a change in values?
D. Ask several managers to explain
A. Values are neither stable nor how their behavior is consistent with
enduring. the organizational culture.

B. The process of questioning values


will result in a change. [156] Source: CIA 0596 II-32
The most effective way for a manager to
C. Values are not fixed, and when they delegate a task to a staff member would be
change, they change quickly. to

D. Values are established in early years A. Define the desired outcome and the
and are unlikely to change. approach precisely and in writing.

B. Define the desired outcome


[153] Source: CIA 0596 II-33 precisely, discuss possible approaches
A manager discovers by chance that a newly with the staff auditor, and reach
hired auditor has strong beliefs that are very agreement on the approach to be taken.
different from the manager's and from those
of most of the audit staff. The manager's best C. Let the staff member try to perform
course of action would be to the task for a defined period of time and
then meet to critique the approach,
A. Facilitate the reassignment of the clarifying the assignment as needed.
new hire as quickly as possible before
this situation becomes disruptive. D. Give the assignment in very general
terms, have the staff member develop
B. Ask the rest of the team for their the desired outcome and approach, and
reaction and act according to the group then review and critique the staff
consensus. member's decisions.

C. Take no action unless the new hire's


behavior is likely to cause harm to the [157] Source: CIA 1196 II-26
organization. If a supervisor uses a supportive
management approach, evidenced by
D. Try to counsel the new hire into positive feelings and concern for
more reasonable beliefs. subordinates, a problem might result
because

[154] Source: CIA 1196 II-37 A. An approach based on pure power


Which of the following is not an advantage makes it difficult to motivate staff.
As a conflict resolution strategy, optimizing
B. This approach depends on material (or a win-win strategy) is most appropriate
rewards for the worker. when

C. This approach depends on people A. The benefits being contested cannot


who want to work, grow, and achieve. be changed.

D. The manager must believe in the B. The relationship between the parties
teamwork approach. is likely to continue.

C. People are deeply committed to


[158] Source: CIA 1196 II-27 established habits and patterns.
Some behavioral models stress employee
participation as a key to motivation. A D. Time is scarce and the manager's
limitation of the participative approach is patience is wearing thin.

A. Workers are intrinsically lazy and


must be driven. [162] Source: CIA 1196 II-19
The behavioral science literature identifies
B. A number of dissatisfiers must be diffusion as an effective approach to
present in order for the approach to resolving conflict. A manager effectively
work. using diffusion in working with a
confrontational employee would
C. It is difficult to elicit the
participation of all employees. A. Set aside critical issues temporarily
and try to reach agreement on less
D. Unresolvable conflicts arise when a controversial issues first.
mature, capable, creative person joins a
structured, demanding, and limiting B. Emphasize differences between the
organization. parties.

C. Avoid the conflict situation.


[159] Source: CIA 1196 II-32
A production worker in a plant often speaks D. Identify the sources of conflict and
for the entire workforce when problems address them directly.
arise between labor and management.
Although this individual has the same level
of authority and expertise as his co-workers, [163] Source: CIA 1196 II-33
he seems to possess a degree of power that During a performance appraisal, the
others do not have. What type of power does manager experienced difficulty obtaining
this individual apparently have? required information from a specific
employee. The manager requested a private
A. Coercive. meeting with the employee for the purpose
of identifying the problem and resolving the
B. Referent. difficulty through open discussion. Which
conflict management technique was the
C. Expert. auditor applying?

D. Legitimate. A. Problem solving.

B. Expansion of resources.
[160] Source: CIA 1196 II-34
A manager believes that positive C. Authoritative command.
reinforcement is the most appropriate way
to deal with professional staff of the type D. Altering the human variable.
working in his/her department. Which of the
following actions demonstrates the principle
of positive reinforcement? [164] Source: CIA 0596 II-27
A worker receives outstanding performance
A. An employee is given a 2-day evaluations from his first three supervisors.
suspension without pay if errors exceed He tries hard to please his fourth supervisor,
a predefined level. but the more he accomplishes, the more
critical his new supervisor becomes. The
B. Employees are praised when the new supervisor is generally considered a
detected error rate in their work stays marginal performer. Which of the following
below a pre-defined level. is the most likely source of the problem?

C. Time budgets, which have forced A. The supervisor's insecurity.


employees to rush and consequently
make errors, are eliminated. B. The worker's performance.

D. Employees are not required to work C. The work environment.


overtime if errors detected in their
work stay below a predefined level. D. The job standards.

[161] Source: CIA 0596 II-22 [165] Source: CIA 1196 II-28
Which of the following criteria for C. Inventory increases or decreases.
measuring the quality of employee
performance would be appropriate for use D. Changes in fixed manufacturing
with a group of professionals, such as a overhead.
college faculty?

I. Quantity of output [Fact Pattern #6]


II. Quality of output GWW Company incurred the following
III. Cost manufacturing costs in its first month of
IV. Timeliness of output operations.
V. Capital requirements
VI. Revenue produced Raw materials $10,000
A. I, II, and III only. Direct labor 20,000
Variable factory overhead 20,000
B. I, II, and IV only. Fixed factory overhead 30,000
Production totaled 2,000 units. Sales amounted to 1,900
C. III, V, and VI only. units at a
price of $100 each.
D. I, II, III, IV, V, and VI.
[170] Source: Publisher
(Refers to Fact Pattern #6)
[166] Source: CIA 1196 II-30 The ending inventory under the variable
Which of the following is not a good costing method is
example of a way to evaluate the quality of
an individual's work? A. $1,500.

A. Evaluate the task outcome to B. $2,500.


determine if the job gets done.
C. $4,000.
B. Use multiple evaluators to increase
reliability.
D. $10,000.
C. Use multiple criteria because every
employee is expected to do a number of
things well. [171] Source: Publisher
(Refers to Fact Pattern #6)
D. Evaluate the worker's traits, such as The ending inventory under the absorption
attitude, intelligence, or reliability. costing method is

A. $1,500.
[167] Source: Publisher
Net income is lower under variable costing B. $2,500.
than under absorption costing when
C. $4,000.
A. Production increases from the
previous period. D. $10,000.

B. Production exceeds sales.


[172] Source: Publisher
C. Production equals sales. (Refers to Fact Pattern #6)
Absorption costing profits will exceed
D. Production is less than sales. variable costing profits by

A. $1,500.
[168] Source: Publisher
If a company uses variable costing instead B. $4,000.
of absorption costing, it will always report
a C. $30,000.

A. Higher income. D. $0.

B. Lower income.
[173] Source: Publisher
C. Higher finished goods inventory. If inventory quantities increase during a
period,
D. Lower finished goods inventory.
A. Variable costing profits will exceed
absorption costing profits.

[169] Source: Publisher B. Absorption costing profits will


Under variable costing, profit fluctuates in exceed variable costing profits.
relation to
C. Variable costing profits will equal
A. Sales. absorption costing profits.

B. Production. D. Variable costing will show a higher


inventory value than absorption costing.
A. Variable manufacturing overhead
manufacturing overhead costs.
[174] Source: Publisher
Advocates of the variable costing method B. Fixed manufacturing overhead costs.
believe that fixed overhead costs should be
expense as incurred because C. All overhead costs.

A. Product costs will occur regardless D. All manufacturing costs.

of production levels.
[179] Source: Publisher
B. Allocation of fixed production costs If unit costs remain unchanged and sales
is arbitrary. volume and sales price per unit both
increase from the preceding period when
C. Production costs are uncontrollable operating profits were earned, operating
and should not be charged to profits must
production.
A. Increase under the absorption costing
D. Period costs are not necessary to method.
produce a product.
B. Increase under the variable costing
method.
[175] Source: Publisher
Although the absorption costing method is C. Decrease under the absorption
ordinarily required by GAAP, a company
may also want to prepare statements under costing method.
the variable costing method to
D. Decrease under the variable costing
A. Report a greater income. method.

B. Better evaluate the activities of


production managers. [180] Source: Publisher
Which of the following is NOT an
C. Better evaluate sales personnel. advantage of using variable costing for
internal reporting purposes?
D. Reduce income taxes.
A. Fixed costs are reported at incurred
values, not absorbed values, thus
[176] Source: Publisher improving control over those costs.
Under the variable costing method of cost
allocation, B. Profits are directly influenced by
changes in sales volume.
A. Variable manufacturing overhead is
treated as a period cost. C. The impact of fixed costs on profits
is emphasized.
B. Variable manufacturing overhead is
expensed as incurred. D. Total costs may be overlooked when
evaluating profits.
C. Variable direct labor costs are
expensed as period costs.
[181] Source: Publisher
D. Variable manufacturing overhead is Which of the following is an advantage of
inventoried as a production cost. the variable costing method?

A. Marginal contribution is emphasized.

[177] Source: Publisher B. The distinction between fixed and


As compared with total absorption costing variable costs is often too difficult to
profit over the entire life of a company, total determine in practice.
variable costing profit will
C. Financial statements might confuse
A. Be less. users who are accustomed to absorption
costing statements.
B. Be greater.
D. Emphasis on variable costs may
C. Be equal. cause managers to ignore fixed costs.

D. Be substantially greater or less


depending upon external factors. [182] Source: Publisher
Contribution margin is defined as the
difference between
[178] Source: Publisher
Absorption costing inventories all A. Variable costing and absorption
manufacturing overhead costs. Variable costing net income.
costing expenses which of the following as
period costs? B. Sales and total variable costs.
C. Fixed and variable manufacturing [187] Source: Publisher
costs. Sheila is a financial manager who has
discovered that her company is violating
environmental regulations. If her immediate
D. Sales and fixed manufacturing costs. superior is involved, her appropriate action
is to

[183] Source: Publisher A. Do nothing since she has a duty of


A company's flexible budget shows an loyalty to the organization.
expected variable delivery expense of
$160,000 when sales are 50,000 units. If B. Consult the audit committee.
sales total 52,000 units, and the actual
delivery expense is $163,000, what will be C. Present the matter to the next higher
the given sales variance? managerial level.

A. $3,000 unfavorable. D. Confront her immediate superior.

B. $3,400 unfavorable.
[188] Source: Publisher
C. $3,000 favorable. Corporate social responsibility is

D. $3,400 favorable. A. Effectively enforced through the


controls envisioned by classical
economics.
[184] Source: Publisher
A company's flexible budget shows an B. Defined as the obligation to
expected fixed cost of $100,000 for shareholders to earn a profit.
straight-line depreciation when sales total
50,000 units. If sales total 52,000 units, and C. More than the obligation to
the actual cost of depreciation is $103,000, shareholders to earn a profit.
what will be the budget variance?
D. Defined as the obligation to serve
A. $1,000 favorable. long-term, organizational interests.

B. $3,000 favorable.
[189] Source: Publisher
C. $1,000 unfavorable.
A common argument against corporate
D. $3,000 unfavorable. involvement in socially responsible
behavior is that

[185] Source: CMA Samp Q3-12 A. It encourages government intrusion


To meet Zylon Corporation's overall in decision making.
objectives, the Frame Division has just
initiated a program to increase sales by B. As a legal person, a corporation is
improving the manufacturing quality of its accountable for its conduct.
products. The most appropriate management
level to be responsible for this program is C. It creates goodwill.
the
D. In a competitive market, such
A. Sales manager. behavior incurs costs that place the
company at a disadvantage.
B. Production manager.

C. Division president. [190] Source: Publisher


The IMA Code of Ethics requires a financial
D. Receiving and inspection manager. manager/management accountant to follow
the established policies of the organization
when faced with an ethical conflict. If these
[186] Source: Publisher policies do not resolve the conflict, the
If a financial manager/management financial manager/management accountant
accountant has a problem in identifying should
unethical behavior or resolving an ethical
conflict, the first action (s)he should A. Consult the board of directors
normally take is to immediately.

A. Consult the board of directors. B. Discuss the problem with the


immediate superior if (s)he is involved
B. Discuss the problem with his/her in the conflict.
immediate superior.
C. Communicate the problem to
C. Notify the appropriate law authorities outside the organization.
enforcement agency.
D. Contact the next higher managerial
D. Resign from the company. level if initial presentation to the
immediate superior does not resolve the
conflict.
C. The financial manager/management
accountant knowingly communicates the
[191] Source: Publisher information indirectly through a
Financial managers/management accountants subordinate.
are obligated to maintain the highest
standards of ethical conduct. Accordingly, D. An officer at the financial
the IMA Code of Ethics explicitly requires manager/management accountant's bank
that they has requested information on a
transaction that could influence the
A. Obtain sufficient competent evidence firm's stock price.
when expressing an opinion.

B. Not condone violations by others.

C. Comply with generally accepted


auditing standards.

D. Adhere to generally accepted


accounting principles.

[192] Source: Publisher


Integrity is an ethical requirement for all
financial managers/management accountants.
One aspect of integrity requires

A. Performance of professional duties


in accordance with applicable laws.

B. Avoidance of conflict of interest.

C. Refraining from improper use of


inside information.

D. Maintenance of an appropriate level


of professional competence.

[193] Source: Publisher


A financial manager/management accountant
discovers a problem that could mislead
users of the firm's financial data and has
informed his/her immediate superior. (S)he
should report the circumstances to the audit
committee and/or the board of directors only
if

A. The immediate superior, who reports


to the chief executive officer, knows
about the situation but refuses to correct
it.

B. The immediate superior assures the


financial manager/management
accountant that the problem will be
resolved.

C. The immediate superior reports the


situation to his/her superior.

D. The immediate superior, the firm's


chief executive officer, knows about the
situation but refuses to correct it.

[194] Source: Publisher


In which situation is a financial
manager/management accountant permitted
to communicate confidential information to
individuals or authorities outside the firm?

A. There is an ethical conflict and the


board has refused to take action.

B. Such communication is legally


prescribed.
CMA PART 3A
BEHAVIOURAL ISSUES [3] Source: CMA 0691 3-24
ANSWERS Answer (A) is correct. The ROI
calculation divides return (income) by
invested capital. Invested capital may
[1] Source: Publisher be defined in various ways, for
example, as total assets available, total
Answer (A) is incorrect because a assets employed (assets that are idle,
top-down budget is generated by top such as vacant land, are excluded),
management and distributed to working capital plus other assets
(imposed on) lower-level managers. (current liabilities are deducted from
total assets to exclude the assets
Answer (B) is incorrect because a provided by short-term creditors), and
bottom-up budget is generated by stockholders' equity (a portion of
lower-level management and long-term liabilities must be allocated
aggregated as it moves through the chain to determine the manager's resource
of command. base). Total assets available is the
measure of invested capital that
Answer (C) is correct. The management assumes the manager will use all assets
by objectives (MBO) approach is a without regard to financing.
procedure in which a subordinate and a
supervisor agree on goals and the Answer (B) is incorrect because use of
methods of achieving them and develop total assets employed assumes that
a plan in accordance with that higher-level management has not
agreement. The subordinate is then required the manager to assume
evaluated with reference to the plan at responsibility for idle assets.
the end of the plan period.
Answer (C) is incorrect because
Answer (D) is incorrect because the deducting current liabilities effectively
management by exception approach excludes assets financed by short-term
uses measurable standards and credit.
deviations therefrom to determine when
management action is needed. Answer (D) is incorrect because
shareholders' equity is determined by
deducting total liabilities from total
[2] Source: CMA 1292 3-30 assets. Thus, this measure of the
investment base also considers the issue
Answer (A) is incorrect because of financing.
reciprocal cost allocation does nothing
to enhance goal congruence.
[4] Source: CMA 0691 3-29
Answer (B) is incorrect because
Answer (A) is incorrect because total
reciprocal cost allocation, zero-base assets employed does not reflect
budgeting, and standard costing are not financing decisions.
designed to enhance goal congruence.
Answer (B) is incorrect because
Answer (C) is incorrect because deducting current liabilities from
imposed budgeting and activity-based current assets to arrive at working
costing are not designed to enhance goal capital reflects the control that the
congruence. manager of the responsibility center
ordinarily has over short-term credit
Answer (D) is correct. Transfer prices transactions.
based on cost promote goal congruence
by ensuring that purchases are made at Answer (C) is incorrect because total
the lowest cost for the entity as a whole. assets available does not reflect
Management-by-objective (MBO) financing decisions.
performance evaluation can also be a
goal congruence tool. MBO is a Answer (D) is correct. Stockholders'
behavioral, communications-oriented, equity equals total assets minus total
responsibility approach to employee liabilities. The latter include short-term
self direction. MBO is based on the liabilities incurred at operating levels
philosophy that employees want to of the organization and long-term
work hard if they only know what is liabilities resulting from financing
expected, that employees like to know decisions made by top management.
what their job actually is, and that Accordingly, the investment base used
employees are capable of self direction to measure the performance of a
and self motivation. The key is to manager may reflect the incurrence of
coordinate managers' goals with the liabilities over which (s)he had no
overall goals of the organization. control. A second problem is that the
Participation in budgeting by those allocation of long-term liabilities
affected likewise encourages goal among divisions or segments may be
congruence because those who take part somewhat arbitrary.
in the budget process are likely to
support the outcome.
[5] Source: CMA 1291 3-9 performance report showing actual
costs incurred against budgeted costs
Answer (A) is incorrect because an permits evaluation of a manager and the
investment center has authority not only area for which (s)he is responsible.
over costs and revenues, but also
capital invested. Answer (D) is incorrect because
activity-based costing is a means of
Answer (B) is correct. An investment allocating overhead.
center is responsible for revenues,
expenses, and invested capital. Return
on investment is usually the key [8] Source: CMA 0694 3-28
performance measure of an investment
center. Answer (A) is correct. A well-designed
responsibility accounting system
Answer (C) is incorrect because an establishes responsibility centers within
investment center also has authority an organization. Managerial
over revenues and invested capital. performance should be evaluated only
on the basis of those factors
Answer (D) is incorrect because a controllable by the manager. Managers
service center supports other may control revenues, costs, and/or
organizational units. investment activities. The responsibility
system should induce management
performance that adheres to overall
[6] Source: CMA 1292 3-24 company objectives. Charging the costs
of a rush order to the sales manager
Answer (A) is incorrect because an who authorized the job creates an
investment base is need to calculate incentive for that individual to minimize
residual income. such costs.

Answer (B) is correct. The residual Answer (B) is incorrect because


income method calculates the excess of functional accounting accumulates costs
the return on an investment over a and assets for each service provided or
targeted amount equal to an imputed function performed, without necessarily
interest charge on invested capital. The assigning responsibility for such costs.
rate used is usually the weighted
average cost of capital. Residual Answer (C) is incorrect because a
income may be preferable to ROI transfer-pricing system charges one
because an enterprise will benefit from segment of an organization for goods
expansion as long as residual income is and services that are provided by
earned. Using a ROI, expansion might another segment within the organization.
be rejected if it lowered ROI even
though residual income would increase. Answer (D) is incorrect because
Thus, the residual income method contribution accounting emphasizes
promotes the congruence of a manager's variable costs and their relationship
goal with those of the enterprise. with revenues, but disassociates fixed
Actions that tend to benefit the company
will also tend to improve the measure costs from the departments responsible.
of the manager's performance.

Answer (C) is incorrect because ROI is [9] Source: CMA 1294 3-22
more widely used than residual income.
Answer (A) is incorrect because the
Answer (D) is incorrect because both manager of an assembly line is likely to
measures consider the same items. be responsible for the salaries of
supervisors, which is to some degree
controllable by the manager.
[7] Source: CMA 0694 3-27
Answer (B) is incorrect because the
Answer (A) is incorrect because manager of an assembly line is likely to
transfer prices are amounts charged by be responsible for the materials, which
one segment of an organization for is to some degree controllable by the
goods or services provided to another manager.
segment.
Answer (C) is incorrect because the
Answer (B) is incorrect because manager of an assembly line is likely to
flexible budgeting is simply a series of be responsible for the repairs and
budgets for varying levels of activity. maintenance, which is to some degree
controllable by the manager.
Answer (C) is correct. A well-designed
responsibility accounting system Answer (D) is correct. Responsibility
establishes responsibility centers within accounting holds managers responsible
an organization. Managerial only for factors under their control. The
performance should be evaluated only depreciation of equipment will
on the basis of those factors probably not appear on the performance
controllable by the manager. Managers report of an assembly-line manager
may control revenues, costs, and/or because the manager usually has no
investment activities. A departmental control over the investment in the
equipment.
Answer (C) is incorrect because
increasing sales volume while holding
[10] Source: Publisher other factors constant improves return
on investment.
Answer (A) is incorrect because dollar
sales do not give a measure of operating Answer (D) is correct. ROI equals net
performance based on resources income divided by investment.
required. Increasing net income (e.g., by
decreasing expenses or by increasing
Answer (B) is incorrect because net prices or sales volume) or decreasing
income does not give a measure of the investment base improves ROI.
operating performance based on Hence, any of the actions listed
resources required. increases the return on investment.
Management and the accounting
Answer (C) is incorrect because profit profession are very concerned with
percentages do not give a measure of classification of expenses and assets
operating performance based on and other decisions involving the
resources required. accounting for these items to achieve a
proper calculation of return on
Answer (D) is correct. Each investment investment.
center of a business should be evaluated
based upon return on investment to
judge operating performance. ROI is [13] Source: CMA 0684 4-9
comparable to calculations made both
within and without a particular Answer (A) is incorrect because ROI
organization. Management may review (NI ÷ I) will decrease if investment
the investment opportunities available (total assets) increases while NI
within or without the firm. In essence, remains the same.
net income is stated as a proportion of
investment capital (resources required). Answer (B) is correct. The basic
formula for return on investment is (S ÷
I) x (NI ÷ S) = NI ÷ I. If NI increases
[11] Source: Publisher (because expenses have decreased) by
the same amount that I increases
Answer (A) is incorrect because I ÷ NI (because total assets have increased),
is the reciprocal of return on ROI will increase. For example, if NI
investment. equals 100 and I equals 1,000, the ROI
is 10%. An increase of 20 in each term
Answer (B) is correct. The first term in raises the ROI to 11.8%.
the formula for return on investment is
the number of times investment capital Answer (C) is incorrect because a
has turned over through the sales reduction in NI causes a decrease in
mechanism (S ÷ I). This amount is ROI if investment is constant.
multiplied by the net income expressed
as a percentage of sales (NI ÷ S) to give Answer (D) is incorrect because ROI
the return on investment (NI ÷ I). The (NI ÷ I) does not change if the company
basic formula for return on investment originally had a net profit and the net
is therefore profit on sales (sales - expenses)
increases by the same percentage as the
(S ÷ I) x (NI ÷ S) = NI ÷ I. total assets.

Answer (C) is incorrect because sales [14] Source: Publisher


must be divided by investment (S ÷ I) to
determine the number of times Answer (A) is incorrect because equal
investment capital has turned over, goals should not be set owing to
which results in NI ÷ I as return on differences in competitive environment,
investment. the strategic goals of the firm, and risk.

Answer (D) is incorrect because net Answer (B) is incorrect because using
income must be expressed as a greater divisional ROIs in the less
percentage of sales (NI ÷ S), which profitable divisions to motivate those
results in NI ÷ I as return on investment. divisions to achieve higher ROIs would
not necessarily improve divisional
performance.
[12] Source: Publisher
Answer (C) is incorrect because lower
Answer (A) is incorrect because divisional ROIs in more profitable
decreasing expenses or assets while divisions in which motivation is
holding other factors constant improves unnecessary would likely suboptimize
return on investment. divisional performance.

Answer (B) is incorrect because Answer (D) is correct. Each division


increasing selling prices while holding within a firm should have an ROI based
other factors constant improves return on the strategic goals of the firm
on investment. consistent with its competitive
environment. some appropriate amount.

Answer (D) is incorrect because


[15] Source: Publisher different prices recorded by the buying
and selling divisions is characteristic of
Answer (A) is incorrect because a dual-pricing policy.
motivation is the desire and the
commitment to achieve a specific goal.
[18] Source: Publisher
Answer (B) is correct. Goal congruence
is agreement on the goals of the Answer (A) is incorrect because the
organization and/or the segment by both market price is the price on the open
supervisors and subordinates. market.
Performance is assumed to be
optimized when there is an Answer (B) is incorrect because the
understanding that personal and outlay cost plus opportunity cost is the
segmental goals be consistent with price representing the cash outflows of
those of the organization. the supplying division plus the
contribution to the supplying division
Answer (C) is incorrect because from an outside sale.
autonomy is the extent to which
individuals have the authority to make Answer (C) is correct. Full-cost price
decisions. is the price usually set by an
absorption-costing calculation and
Answer (D) is incorrect because includes materials, labor, and a full
managerial effort is the extent of the allocation of manufacturing O/H. This
attempt to accomplish a specific goal. full-cost price may lead to
dysfunctional behavior by the supplying
and receiving divisions, e.g.,
[16] Source: Publisher purchasing from outside sources at a
slightly lower price that is substantially
Answer (A) is incorrect because the above the variable costs of internal
retail price is the definition of the production.
market price, assuming an arm's-length
transaction. Answer (D) is incorrect because the
variable-cost-plus price is the price set
Answer (B) is correct. At this price, the by charging for variable costs plus a
supplying division is indifferent as to lump sum or an additional markup, but
whether it sells internally or externally. less than full markup.
Outlay cost plus opportunity cost
therefore represents a minimum
acceptable price for a seller. However, [19] Source: CIA 1190 IV-20
no transfer price formula is appropriate
in all circumstances. Answer (A) is incorrect because
inefficiencies are charged to the buying
Answer (C) is incorrect because full department.
cost is the price usually set by an
absorption-costing calculation. Answer (B) is correct. The optimal
transfer price of a selling division
Answer (D) is incorrect because the should be set at a point that will have
variable-cost-plus price is the price set the most desirable economic effect on
by charging for variable costs plus a the firm as a whole while at the same
lump sum or an additional markup, but time continuing to motivate the
less than full markup. management of every division to
perform efficiently. Setting the transfer
price based on actual costs rather than
[17] Source: Publisher standard costs would give the selling
division little incentive to control costs.
Answer (A) is incorrect because the
price on the open market is the Answer (C) is incorrect because, by
definition of the market price. definition, cost-based transfer prices
are not adjusted by a markup or
Answer (B) is incorrect because outlay markdown.
cost plus opportunity cost is the price
representing the cash outflows of the Answer (D) is incorrect because
supplying division plus the contribution cost-based transfer prices provide the
to the supplying division from an advantages of clarity and administrative
outside sale. convenience.

Answer (C) is correct. The


variable-cost-plus price is the price set [20] Source: CIA 1183 IV-5
by charging for variable cost plus either
a lump sum or an additional markup but Answer (A) is incorrect because
less than the full markup price. This outside purchase will increase the
permits top management to enter the company's cost of sales by $10 per unit.
decision process and dictate that a
division transfer at variable cost plus Answer (B) is incorrect because the
transfer price is irrelevant to the [23] Source: CIA 0588 IV-19
decision. It does not affect overall
profits. Answer (A) is incorrect because the
alpha division has no idle capacity.
Answer (C) is incorrect because the
company is initially concerned with Answer (B) is incorrect because the
covering variable rather than fixed opportunity cost needs to be included.
costs.
Answer (C) is incorrect because the
Answer (D) is correct. Opportunity minimum transfer price equals outlay
costs are $0 because A's facilities (variable) costs plus opportunity cost,
would be idle if B did not purchase not variable costs plus fixed costs.
from A. Assuming fixed costs are not
affected by the decision, the Answer (D) is correct. In a perfectly
intracompany sale is preferable from competitive market, market price is
the company's perspective because A's ordinarily the appropriate transfer
$30 variable unit cost is less than the price. Because the market price is
outside vendor's price of $40. objective, using it avoids waste and
maximizes efficiency. In a perfectly
competitive market, the market price
[21] Source: CIA 0592 IV-19 equals the minimum transfer price,
which is the sum of outlay cost and
Answer (A) is incorrect because $12 opportunity cost. Outlay cost is the
per unit merely allows Division Z to variable cost per unit, or $34 ($34,000
recover its unit variable cost. ÷ 1,000). Opportunity cost is the
contribution margin foregone, or $16
Answer (B) is correct. A unit price of ($50 - $34). Thus, the minimum transfer
$18 is less than Division Y's cost of price is $50 ($34 + $16).
purchase from an outside supplier but
exceeds Division Z's production cost.
Accordingly, both Y and Z benefit. [24] Source: CIA 0593 IV-16

Answer (C) is incorrect because, at $20 Answer (A) is incorrect because


per unit, Division Y may be indifferent internally developed information should
as to whether it purchases internally or be developed whether or not transfer
externally. Buying from an outside prices are used.
source for $20 per unit is contrary to the
company's interests given idle capacity Answer (B) is incorrect because market
available for the component's effects on company operations are
manufacture and an incremental unit characteristic of a market-based
cost of $20. transfer price.

Answer (D) is incorrect because, at $22 Answer (C) is incorrect because


per unit, Division Y would have externally developed information is
incentive to purchase from an external needed for a market-based transfer
supplier (i.e. market price is $20). price.

Answer (D) is correct. A cost-based


[22] Source: CIA 0589 IV-16 transfer price is a price charged in an
intracompany transaction that covers
Answer (A) is correct. From the seller's only the selling subunit's costs.
perspective, the price should reflect at However, by ignoring relevant
least its incremental cash outflow alternative market prices, a company
(outlay cost) plus the contribution from may pay more than is necessary to
an outside sale (opportunity cost). produce goods and services internally.
Because A has idle capacity, the
opportunity cost is $0. Thus, the
minimum price Division A should [25] Source: CIA 1193 IV-18
charge Division B is $7.00.
Answer (A) is incorrect because
Answer (B) is incorrect because $7.00 marginal production cost transfer prices
is the minimum that should be charged. do not relate to market-based transfer
Since Division A has idle capacity, the prices.
minimum transfer price should recover
variable costs ($7.00). Answer (B) is incorrect because
marginal cost based transfer prices
Answer (C) is incorrect because provide more of an incentive to the
Division A should not include any fixed purchasing division to buy internally
costs in their transfer price because and thus use idle facilities of the selling
Division A has idle capacity. division than does the usually higher
market-based transfer price.
Answer (D) is incorrect because, since
Division A has idle capacity, the Answer (C) is correct. A transfer price
minimum transfer price should recover is the price charged in an intercompany
Division A's variable (outlay) costs. transaction. Market-based prices
provide market discipline because
efficient internal suppliers will tend to
prosper, thereby enhancing the overall calculating transfer prices for a firm
long-term competitiveness of the firm. may correctly price the firm's product in
Country A but not in Country B. The
Answer (D) is incorrect because product may be overpriced in Country
corporate politics is less of a factor B, causing sales to be lower than
than in other methods, such as a anticipated. Alternatively, the product
negotiated transfer price. Market-based may be underpriced in Country B, and
prices are objective. the authorities may allege that the firm
is dumping its product there.

[26] Source: CIA 1193 IV-19


[28] Source: CIA 0594 III-71
Answer (A) is correct. A transfer price
is the price charged by one segment of Answer (A) is incorrect because
an organization for a product or service responsibility accounting holds
supplied to another segment of the same managers responsible only for what
organization. The three basic criteria they can control.
that the transfer pricing system in a
decentralized company should satisfy Answer (B) is incorrect because a cost
are to (1) provide information allowing center manager is concerned with costs
central management to evaluate only, whereas a profit center manager is
divisions with respect to total company concerned with costs and revenues.
profit and each division's contribution
to profit, (2) stimulate each manager's Answer (C) is correct. Responsibility
efficiency without losing each accounting stresses that managers are
division's autonomy, and (3) motivate responsible only for factors under their
each divisional manager to achieve control. For this purpose, the operations
his/her own profit goal in a manner of the business are organized into
contributing to the company's success. responsibility centers. Costs are
Because the $220 transfer price classified as controllable and
selected is based on the quoted external uncontrollable. This implies that some
price (market), it is an example of revenues and costs can be changed
market-based transfer pricing. through effective management.
Management may then focus on
Answer (B) is incorrect because the deviations for either reinforcement or
cost-based price would be $210 ($100 correction. Thus, the statement that
+ $50 + $40 + $20). every factor is ultimately controllable
by someone is not a premise of
Answer (C) is incorrect because no responsibility accounting.
negotiations took place.
Answer (D) is incorrect because this is
Answer (D) is incorrect because cost the essence of responsibility
plus 20% would be $252 ($210 x 1.20). accounting. Each manager is held
accountable for factors under their
control.
[27] Source: CIA 0594 III-40

Answer (A) is incorrect because [29] Source: CIA 1191 IV-18


properly chosen transfer prices allow
firms to attempt to minimize worldwide Answer (A) is incorrect because
taxes by producing various parts of the corporate administrative costs should
products in different countries and be excluded from the performance
strategically transferring the parts at report. The segments have no control
various systematically calculated over their incurrence or the allocation
prices. basis. The allocation depends upon the
segment sales (controllable) as well as
Answer (B) is incorrect because the sales of other segments
properly chosen transfer prices allocate (uncontrollable).
revenues and expenses to divisions in
various countries. These numbers are Answer (B) is incorrect because the
used as part of the input for the segments have no control over the
performance evaluation of each incurrence of personnel costs or the
division. method of assignment, which depends
upon the number of employees in the
Answer (C) is incorrect because segment (controllable) in proportion to
transfer prices motivate division the total number of employees in all
managers to buy parts and products segments (not controllable).
(from either internal or external
suppliers) at the lowest possible prices Answer (C) is incorrect because the
and to sell their products (to either segments have no control over fixed
internal or external customers) at the computer facility costs, and the equal
highest possible prices. Hence, each assignment is arbitrary and bears no
division has a profit making orientation. relation to usage.

Answer (D) is correct. The calculation Answer (D) is correct. The variable
of transfer prices in the international computer cost can be included. The
arena must be systematic. A scheme for segments are charged for actual usage,
which is under each segment's control. revenues and expenses (s)he can
The predetermined standard rate is set control. Controllability is the basic
at the beginning of the year and is concept of responsibility accounting.
known by the segment managers.
Moreover, the efficiencies and
inefficiencies of the computer [32] Source: Publisher
department are not passed on to the
segments. Both procedures promote a Answer (A) is correct. A responsibility
degree of control by the segments. accounting system should have certain
controls that provide for feedback
reports indicating deviations from
[30] Source: CIA 1192 IV-22 expectations. Management may then
focus on those deviations (exceptions)
Answer (A) is incorrect because for either reinforcement or correction.
contribution margin reporting separates
costs by behavior; variable costs are Answer (B) is incorrect because the
listed first followed by fixed costs. responsibility accounting system should
Some responsibility accounting systems not be used exclusively to assess blame.
use a contribution margin reporting
format, but contribution margin Answer (C) is incorrect because the
reporting alone can include costs not responsibility accounting system should
controllable by a manager. not be used exclusively to give
rewards.
Answer (B) is incorrect because
segment reporting is preparation of Answer (D) is incorrect because
performance reports by reportable feedback reports concentrate on
segments. Segment reports often include deviations, but not to the total exclusion
allocated costs that are not controllable of other budgeted variables.
by managers.

Answer (C) is incorrect because [33] Source: Publisher


absorption cost accounting is
characterized by its treatment of fixed Answer (A) is incorrect because
manufacturing overhead as a product generally accepted accounting
cost. principles concern external financial
reporting, not internal reporting.
Answer (D) is correct. Responsibility
accounting stresses that managers Answer (B) is incorrect because the
should only be held responsible for Financial Accounting Standards Board
factors under their control. To achieve concerns external financial reporting,
this objective, the operations of the not internal reporting.
business are broken down into
responsibility centers. Costs are Answer (C) is incorrect because the
classified as controllable and American Institute of Certified Public
noncontrollable to assign responsibility. Accountants concerns external financial
The assignment of responsibility reporting, not internal reporting.
implies that some revenues and costs
can be changed through effective Answer (D) is correct. The
management. A responsibility responsibility for internal reports is
accounting system should have certain management's. Management may direct
controls that provide for feedback the accountant to provide a report in any
reports indicating deviations from format deemed suitable for the decision
expectations. Management may then process. The accountant should work
focus on those deviations for either closely with management to make these
reinforcement or correction. reports an effective communication
device regarding the firm and its
decisions.
[31] Source: CIA 0589 IV-15

Answer (A) is incorrect because all [34] Source: CIA 1191 IV-17
variable costs may not be controllable,
but some, if not all, fixed costs might be Answer (A) is correct. The
controllable. responsibility accounting report should
list only the costs over which the
Answer (B) is incorrect because not all warehousing supervisor exercises
budgeted costs are controllable by control. The supervisor's salary should
managers. therefore be excluded because it is
controlled by the warehouse
Answer (C) is incorrect because all supervisor's superior. Moreover, only
product costs may not be controllable, the product costs are to be considered.
but some, if not all, period costs might These exclude the shipping clerks'
be controllable. wages and fringe benefits because they
are period costs (shipping is a selling
Answer (D) is correct. The most expense). Thus, the only product cost
desirable measure for evaluating a under the control of the warehouse
departmental manager is one that holds supervisor is the receiving clerks'
the manager responsible for the wages ($75,000) and the related fringe
benefits (.3 x $75,000 = $22,500), or a
total of $97,500.
[37] Source: Publisher
Answer (B) is incorrect because
$128,700 equals 130% of the sum of Answer (A) is incorrect because net
60% of the supervisor's salary, and income as a percentage of sales is
100% of the receiving clerks wages. calculated by dividing 6% (minimum
ROI) of the investment by the sales of
Answer (C) is incorrect because Segment B.
$130,000 is the sum of the wages of the
receiving and shipping clerks. Answer (B) is incorrect because 6.67%
is the net income as a percentage of
sales for Segment C.
Answer (D) is incorrect because
$169,000 includes the shipping and Answer (C) is correct. Residual income
receiving clerks' wages and their was zero, indicating that net income
employee benefit costs. These should was equal to the minimum ROI. Given a
be treated as period costs. 6% minimum ROI as a percentage of
investment, 6% of the $500,000
investment is $30,000. Sales were
[35] Source: CIA 1190 IV-21 $750,000, so net income ($30,000) is
4% of sales.
Answer (A) is correct. Residual income
is income of an investment center, Answer (D) is incorrect because 10%
minus an imputed interest charge for is the minimum ROI percentage for
invested capital. The theory is that Segment D.
earning an income greater than residual
income indicates that expansion is
desirable. However, comparisons of [38] Source: Publisher
investment centers based on residual
income may be misleading because of Answer (A) is incorrect because 5% is
differences in products, markets, costs, the net income as a percentage of sales
and local conditions. for Segment D.

Answer (B) is incorrect because use of Answer (B) is correct. Net income as a
the same imputed interest rate provides percentage of sales is the ROI divided
a consistent objective against which by turnover of investment. The turnover
each investment can be measured. of the investment is sales ($135,000)
divided by the investment ($45,000), or
Answer (C) is incorrect because 3. Hence, net income is 6.67% (20% ÷
common amounts of invested capital 3) as a percentage of sales for Segment
would eliminate a major factor causing C.
differences in residual income.
Answer (C) is incorrect because 4% is
Answer (D) is incorrect because the net income as a percentage of sales
comparisons of investment centers for Segment B.
based on residual income may be
misleading because of peculiarities of Answer (D) is incorrect because 20%
each investment center (i.e. differences is the ROI for Segment C.
in products, markets, costs, and local
conditions).
[39] Source: Publisher

[36] Source: Publisher Answer (A) is correct. The turnover of


investment for Segment C is calculated
Answer (A) is incorrect because by dividing sales by investment. Given
motivation is the desire and the sales of $135,000 and investment of
commitment to achieve a specific goal. $45,000, Segment C's turnover of
investment is 3.
Answer (B) is incorrect because
autonomy is the extent to which Answer (B) is incorrect because 1.5 is
individuals have the authority to make the turnover of investment for Segment
decisions. B.

Answer (C) is correct. Managerial Answer (C) is incorrect because 2.5 is


effort is the extent to which a manager the turnover of investment for Segment
attempts to accomplish a goal. A.
Managerial effort may include
psychological as well as physical
commitment to a goal. Answer (D) is incorrect because the
turnover of investment is calculated by
Answer (D) is incorrect because dividing sales by investment.
managerial effort reflects the attempts
of an individual manager or
organizational unit, not those efforts [40] Source: Publisher
shared among managers of different
organization units. Answer (A) is correct. The turnover of
investment for Segment D is determined for Segment D.
by dividing sales by investment. For
Segment D, net income ($90,000) as a Answer (D) is incorrect because
percentage of sales ($1,800,000) equals $12,000 results from multiplying sales
5%. ROI is given as 7.5%. Dividing net (rather than investment) by the minimum
income as a percentage of sales (5%) rate of return percentage.
into ROI (7.5%) gives a turnover of
investment of 1.5.
[44] Source: Publisher
Answer (B) is incorrect because 2.5 is
the turnover of investment for Segment Answer (A) is correct. The ROI in
A. dollars is equal to the amount of the
investment times the minimum rate of
Answer (C) is incorrect because the return percentage. The amount of the
turnover of investment is calculated by investment is $500,000. The minimum
dividing sales by investment. rate of return percentage is given as
6%. Thus, the minimum ROI in dollars
Answer (D) is incorrect because 20 equals $30,000.
results from dividing sales by net
income. Answer (B) is incorrect because $6,750
is the minimum dollar ROI for Segment
C.
[41] Source: Publisher
Answer (C) is incorrect because $4,800
Answer (A) is incorrect because ROI is is the minimum dollar ROI for Segment
calculated by dividing net income by A.
investment.
Answer (D) is incorrect because
Answer (B) is correct. ROI is equal to $45,000 results from multiplying sales
net income divided by investment. Net (rather than investment) by the minimum
income equals $5,000. Investment rate of return percentage.
equals $24,000. ROI equals 20.8%.

Answer (C) is incorrect because ROI is [45] Source: Publisher


calculated by dividing net income by
investment. Answer (A) is correct. The minimum
ROI in dollars is equal to the minimum
Answer (D) is incorrect because 8.3% ROI percentage times the investment.
is the net income as a percent of sales The investment was $45,000. Neither
(5,000/60,000). the minimum percentage nor the
minimum ROI is known. However, the
ROI percentage (20%) and the
[42] Source: Publisher investment ($45,000) are known.
Hence, the net income is $9,000. Given
Answer (A) is correct. Residual income residual income of $2,250, the minimum
is given as zero. Thus, the actual ROI is ROI in dollars must have been $6,750
the same as the minimum percentage ($9,000 - $2,250).
ROI of 6%.
Answer (B) is incorrect because $4,800
Answer (B) is incorrect because ROI is is the minimum dollar ROI for Segment
calculated by dividing net income by A.
investment.
Answer (C) is incorrect because
Answer (C) is incorrect because 20% $120,000 is the minimum dollar ROI
is the ROI for Segment C. for Segment D.

Answer (D) is incorrect because 7.5% Answer (D) is incorrect because


is the ROI for Segment D. $9,000 results form multiplying
investment by actual (rather than
minimum) ROI.
[43] Source: Publisher

Answer (A) is incorrect because [46] Source: Publisher


$30,000 is the minimum dollar ROI for
Segment B. Answer (A) is incorrect because 20%
is the minimum ROI percentage for
Answer (B) is correct. The ROI in Segment A.
dollars is equal to the amount of the
investment times the minimum rate of Answer (B) is incorrect because 6% is
return percentage. The amount of the the minimum ROI percentage for
investment is $24,000. The minimum Segment B.
rate of return percentage is 20%.
Accordingly, the minimum ROI in Answer (C) is correct. The minimum
dollars is $4,800. percentage of ROI in Segment C equals
the minimum dollar ROI divided by the
Answer (C) is incorrect because investment. The minimum dollar ROI is
$120,000 is the minimum dollar ROI $6,750. Consequently the minimum
percentage ROI is 15% ($6,750 ÷
$45,000). Answer (C) is incorrect because the
residual income for Segment D is
Answer (D) is incorrect because 10% determined by subtracting the minimum
is the minimum ROI percentage for ROI dollars from net income.
Segment D.
Answer (D) is incorrect because the
residual income for Segment D is
[47] Source: Publisher determined by subtracting the minimum
ROI dollars from net income.
Answer (A) is incorrect because 20%
is the minimum ROI percentage for
Segment A. [50] Source: Publisher

Answer (B) is incorrect because 6% is Answer (A) is incorrect because fixed


the minimum ROI percentage for costs per unit decrease because there
Segment B. are more units to absorb the fixed costs
and variable costs do not change over
Answer (C) is incorrect because 15% the relevant range.
is the minimum ROI percentage for
Segment C. Answer (B) is incorrect because fixed
costs per unit decrease because there
Answer (D) is correct. The minimum are more units to absorb the fixed costs.
percentage of ROI for Segment D is the
minimum ROI in dollars ($120,000) Answer (C) is incorrect because
divided by the investment, which must variable costs per unit do not change
be calculated. The ROI is given as over the relevant range.
7.5%. The net income ($90,000) as a
percentage of sales ($1,800,000) equals Answer (D) is correct. Fixed costs per
5%. The turnover of investment (ROI ÷ unit decrease within the relevant range
net income as a percentage of sales) is of activity as production increases
1.5. Given turnover of 1.5 and sales of because more units are available to
$1,800,000, investment must have been absorb the constant amount of total
$1,200,000. The minimum percentage fixed costs. Unit variable costs are
ROI is $120,000 divided by the assumed to remain the same per unit
$1,200,000 investment, or 10%. over the relevant range.

[48] Source: Publisher [51] Source: Publisher

Answer (A) is correct. Segment A's Answer (A) is incorrect because a cost
residual income is equal to the net center is not responsible for revenues.
income ($5,000) minus the minimum
ROI in dollars. Minimum ROI in Answer (B) is incorrect because the
dollars equals the minimum ROI performance reports of an investment
percentage (20%) times the investment center and a profit center but not a cost
($24,000), or $4,800. Residual income center include controllable revenues.
is therefore $200.
Answer (C) is correct. In investment
Answer (B) is incorrect because centers, managers are responsible for
$12,000 is the minimum ROI percentage all activities, including costs, revenues,
multiplied by sales. and investments. An investment center
is a profit center with significant
Answer (C) is incorrect because control over the amount of capital
$(30,000) is the residual income for invested. This control extends to
Segment D. investments such as receivables and
property, plant, and equipment, as well
Answer (D) is incorrect because the as entry into new markets. A cost
residual income for Segment A is center, for example, a production
determined by subtracting the minimum department, is responsible for costs
ROI dollars from net income. only. A profit center, for example, the
appliance department in a retail store,
is responsible for both revenues and
[49] Source: Publisher expenses.

Answer (A) is incorrect because the Answer (D) is incorrect because


residual income for Segment D is investment centers and profit centers
determined by subtracting the minimum are responsible for revenues.
ROI dollars from net income.

Answer (B) is correct. The minimum [52] Source: Publisher


ROI in dollars is given as $120,000 and
net income is given as $90,000. Thus, Answer (A) is correct. Allocated fixed
residual income is $(30,000) ($90,000 overhead should not be included in
- $120,000). Segment D did not achieve internal reports based on a
its minimum ROI and therefore has a responsibility accounting system
negative residual income. because it cannot be controlled by a
manager of a responsibility center.
Answer (D) is incorrect because the
Answer (B) is incorrect because, in profit center's manager does not control
responsibility accounting, managers are the listed period expenses and therefore
only held responsible for costs they does not control the profit center's
have the authority to control. income.

Answer (C) is incorrect because the


organizational chart, which outlines the [55] Source: Publisher
authority-responsibility chain of a
company, is an integral part of the Answer (A) is incorrect because
responsibility accounting system. suboptimization occurs when a decision
benefits a segment of a company but not
Answer (D) is incorrect because a main the company as a whole.
purpose of internal reports is to show
the variance between actual and Answer (B) is incorrect because
budgeted controllable costs so suboptimization occurs when a decision
corrective action can be taken when and benefits a segment of a company but not
where needed. the company as a whole.

Answer (C) is incorrect because


[53] Source: Publisher suboptimization occurs when a decision
benefits a segment of a company but not
Answer (A) is correct. A responsibility the company as a whole.
center is a subunit of an organization
whose manager is accountable for a Answer (D) is correct. Suboptimization
specified set of activities. Job-order occurs when one segment of a company
costing, process costing, and takes an action that benefits itself but
activity-based costing all may not the firm as a whole.
accumulate their costs by responsibility
centers.
[56] Source: Publisher
Answer (B) is incorrect because
job-order costing may also accumulate Answer (A) is incorrect because the
costs by responsibility centers for ROI equals capital turnover (sales
control purposes. divided by investment) times the profit
margin (net income divided by sales).
Answer (C) is incorrect because
process costing and activity-based Answer (B) is incorrect because the
costing may also accumulate their costs ROI equals capital turnover (sales
by responsibility centers for control divided by investment) times the profit
purposes. margin (net income divided by sales).

Answer (D) is incorrect because Answer (C) is correct. ROI equals


job-order costing, process costing, and capital turnover (sales divided by
activity-based costing may accumulate investment) times the profit margin (net
their costs by responsibility centers. income divided by sales). Therefore,
Andrew's ROI is 25% [($720,000 ÷
$480,000) x ($120,000 ÷ $720,000)].
[54] Source: Publisher
Answer (D) is incorrect because the
Answer (A) is correct. A profit center ROI equals capital turnover (sales
is a segment of a company responsible divided by investment) times the profit
for both revenues and expenses. A margin (net income divided by sales).
profit center has the authority to make
decisions concerning markets
(revenues) and sources of supplies [57] Source: Publisher
(costs). However, the profit center's
manager does not control his/her salary, Answer (A) is correct. Residual income
investment and the resulting costs (e.g., is income of an investment center minus
depreciation of plant assets), or an imputed interest charge for invested
expenses incurred at the corporate capital. Accordingly, Cinder's residual
level. Consequently, profit center No. income is $170,400 [($600,000 sales -
12 is most likely to control the $84,000 $360,000 variable costs - $60,000
contribution margin (sales - variable traceable fixed costs) net income - (8%
costs) but not the other items in the x $120,000 average invested capital)
summarized income statement. imputed interest].

Answer (B) is incorrect because the Answer (B) is incorrect because the
profit center manager does not control imputed interest charge of $9,600 (8% x
depreciation on accommodations $120,000) must be deducted.
($9,600) or the allocated corporate
expenses ($6,000). Answer (C) is incorrect because the
imputed interest charge of $9,600
Answer (C) is incorrect because the should be subtracted from, not added to,
profit center manager does not control net income.
his/her $24,000 salary.
Answer (D) is incorrect because the [60] Source: Publisher
traceable fixed costs must be deducted.
Answer (A) is correct. The breakeven
point in units is total fixed costs divided
[58] Source: Publisher by the unit contribution margin (UCM).
The UCM is the selling price minus
Answer (A) is incorrect because 18% variable costs per unit. Variable costs
is the rate at which Ash imputes interest
on investments. per unit equal $1.50 ($540,000 ÷
360,000 units). Thus, the UCM equals
Answer (B) is incorrect because 25.7% $.90 ($2.40 - $1.50). Dividing the
results from using year 2 year-end $108,000 of fixed costs by the $.90
invested capital instead of average UCM yields a breakeven point of
invested capital. 120,000 units.

Answer (C) is correct. The return on Answer (B) is incorrect because selling
average investment equals net income 110,000 units would result in a loss of
from operations divided by average $.08 per unit.
invested capital. Average invested
capital is $720,000 [($600,000 + Answer (C) is incorrect because selling
$840,000) ÷ 2]. 100,000 units would result in a loss of
$.18 per unit.
Sales (360,000 units x $2.40) $864,000
Minus: Answer (D) is incorrect because selling
Variable costs (540,000) 90,000 units would result in a loss of
???????? $.30 per unit.
Contribution margin $324,000
Minus:
Fixed costs (108,000) [61] Source: Publisher
????????
Net operating income $216,000 Answer (A) is incorrect because
======== contribution margin is sales minus total
Therefore, the return on average variable costs.
investment was 30% ($216,000 ÷
$720,000). Answer (B) is correct. The contribution
margin is sales of $864,000 (360,000
Answer (D) is incorrect because 36% units x $2.40) minus variable costs of
results from using year 1 year-end $540,000, or $324,000.
invested capital instead of average
invested capital. Answer (C) is incorrect because
contribution margin is sales minus total
variable costs.
[59] Source: Publisher
Answer (D) is incorrect because
Answer (A) is correct. Residual income contribution margin is sales minus total
is equal to net operating income minus variable costs.
imputed interest on invested capital.
Net operating income and average
investment were $72,000 and $720,000, [62] Source: Publisher
respectively. The imputed interest rate
is 18%. Thus, Answer (A) is incorrect because
$648,000 is the total costs and expenses
Net operating income $ 72,000 for year 2 based on a production of
Minus: 360,000 hammers.
Imputed interest (18% x $720,000) (129,600)
????????? Answer (B) is correct. Over the
Residual income $ (57,600) relevant range, fixed costs will not
========= fluctuate. The variable cost per unit was
$1.50. Thus, total costs and expenses
Answer (B) is incorrect because the will be
imputed interest is 18% of the average
invested capital of $720,000, not of the Variable ($1.50 x 400,000 units) $600,000
year-end invested capital of $840,000. Fixed 108,000
????????
Answer (C) is incorrect because the Total costs and expenses $708,000
imputed interest of $129,600 (18% x ========

$720,000) needs to be deducted from Answer (C) is incorrect because, within


net operating income. the relevant range, fixed costs will
remain constant at $108,000.
Answer (D) is incorrect because the
imputed interest is 18% of the average Answer (D) is incorrect because,
invested capital of $720,000, not of the within the relevant range, fixed costs
capital at the beginning of the year of will remain constant at $108,000.
$600,000.

[63] Source: CIA 0594 III-44


Answer (A) is incorrect because
Answer (A) is incorrect because flexible budgeting is the process of
$370,000 measures the performance of preparing a series of multiple budgets
the marketing segment manager for varying levels of production or
($950,000 - $430,000 - $150,000). sales.

Answer (B) is incorrect because Answer (B) is incorrect because human


$10,000 includes the allocated costs resource management is the process of
($950,000 - $430,000 - $150,000 - managing personnel.
$250,000 - $110,000).
Answer (C) is correct. Managerial
Answer (C) is incorrect because performance should ideally be
$520,000 is the contribution margin evaluated only on the basis of those
($950,000 - $430,000). factors controllable by the manager.
Managers may control revenues, costs,
Answer (D) is correct. The best or investments in resources. A
measure of the segment's economic well-designed responsibility accounting
performance includes all costs except system establishes responsibility
the fixed manufacturing costs allocated centers within the organization.
to the segment. Thus, the best measure However, controllability is not an
of economic performance is $120,000 absolute basis for establishment of
($950,000 - $430,000 - $150,000 - responsibility. More than one manager
$250,000). may be able to influence a cost, and
responsibility may be assigned on the
basis of knowledge about the incurrence
[64] Source: CMA 1291 3-10 of a cost rather than the ability to
control it. Management by objectives
Answer (A) is incorrect because a (MBO) is a related concept. It is a
service center has no responsibility for behavioral, communications-oriented,
developing markets or selling. responsibility approach to employee
self-direction. Under MBO, a manager
Answer (B) is incorrect because a and his/her subordinates agree upon
production center is engaged in objectives and the means of attaining
manufacturing. them. The plans that result are reflected
in responsibility accounting and in the
Answer (C) is incorrect because a budgeting process.
profit center can choose its markets and
sources of supply. Answer (D) is incorrect because capital
budgeting is a means of evaluating
Answer (D) is correct. A service center long-term investments.
exists primarily and sometimes solely
to provide specialized support to other
units within the organization. Service [67] Source: CMA 1293 3-21
centers are usually operated as cost
centers. Answer (A) is incorrect because fixed
costs may also be controllable, and
some costs not controllable may need to
[65] Source: CMA 0693 3-14 be assigned.

Answer (A) is incorrect because a Answer (B) is incorrect because


profit center is a segment responsible knowledge about the incurrence of a
for both revenues and costs. A profit cost rather than controllability may in
center has the authority to make practice be an appropriate basis for
decisions concerning markets and delegation of responsibility.
sources of supply.
Answer (C) is correct. Managerial
Answer (B) is incorrect because an performance should ideally be
investment center is a responsibility evaluated only on the basis of those
center that is accountable for revenues factors controllable by the manager.
(markets), costs (sources of supply), Managers may control revenues, costs,
and invested capital. and/or investments in resources.
However, controllability is not an
Answer (C) is incorrect because a absolute. More than one manager may
contribution center is responsible for be able to influence a cost, and
revenues and variable costs, but not managers may be accountable for some
invested capital. costs they do not control. In practice,
given the difficulties of determining the
Answer (D) is correct. A cost center is locus of controllability, responsibility
a responsibility center that is may be assigned on the basis of
accountable only for costs. The cost knowledge about the incurrence of a
center is the least complex type of cost rather than the ability to control it.
segment because it has no responsibility Accordingly, a successful system is
for revenues or investments. dependent upon the proper delegation of
responsibility and the commensurate
authority.
[66] Source: CMA 1293 3-17
Answer (D) is incorrect because fixed
costs can be controllable. accounting holds managers responsible
only for factors under their control. For
this purpose, operations are organized
into responsibility centers. Costs are
[68] Source: CMA 0694 3-24 classified as controllable and
noncontrollable, which implies that
Answer (A) is incorrect because a some revenues and costs can be
revenue center is responsible only for changed through effective management.
revenue generation, not for costs or If a manager has authority to incur costs,
capital investment. a responsibility accounting system will
charge those costs to the manager's
Answer (B) is incorrect because a responsibility center.
profit center is responsible for revenues
and costs but not for invested capital.
[71] Source: CMA 0695 3-28
Answer (C) is incorrect because a cost
center is evaluated only on the basis of Answer (A) is incorrect because many
costs incurred. It is not responsible for overhead costs are also controllable.
revenues or invested capital.
Answer (B) is correct. Control is the
Answer (D) is correct. An investment process of making certain that plans are
center is the organizational type most achieving the desired objectives. A
like an independent business because it controllable cost is one that is directly
is responsible for its own revenues, regulated by a specific manager at a
costs incurred, and capital invested. given level of production within a given
The other types of centers do not time span. For example, fixed costs are
incorporate all three elements. often not controllable in the short run.

Answer (C) is incorrect because


[69] Source: CMA 1294 3-20 controllable costs need not be
discretionary. Discretionary costs are
Answer (A) is correct. Responsibility characterized by uncertainty about the
accounting holds managers responsible relationship between input and the
only for factors under their control. For value of the related output; they may or
this purpose, operations are organized may not be controllable.
into responsibility centers. Costs are
classified as controllable and Answer (D) is incorrect because
noncontrollable, which implies that controllable costs are those over which
some revenues and costs can be a manager has control; the manager may
changed through effective management. be knowledgeable and informed about
If a manager has authority to incur costs, costs that (s)he cannot control.
a responsibility accounting system will
charge them to the manager's
responsibility center. [72] Source: CMA 1295 3-5

Answer (B) is incorrect because Answer (A) is correct. A profit center


functional accounting allocates costs to is responsible for both revenues and
functions regardless of responsibility. costs, whereas a cost center is
responsible only for costs.
Answer (C) is incorrect because
reciprocal allocation is a means of Answer (B) is incorrect because a
allocating service department costs. revenue center is responsible only for
revenues, not costs.
Answer (D) is incorrect because
transfer price accounting is a means of Answer (C) is incorrect because a
charging one department for products division can be any type of
acquired from another department in the responsibility center.
same organization.
Answer (D) is incorrect because an
operating unit can be organized as any
[70] Source: CMA 1294 3-21 type of center.

Answer (A) is incorrect because


contribution accounting is a method of [73] Source: CMA 1296 3-16
control in which only variable costs are
matched with revenues. Answer (A) is incorrect because direct
labor is controllable by the production
Answer (B) is incorrect because manager.
cost-benefit accounting is a nonsense
term referring to matching costs and Answer (B) is incorrect because
benefits. materials are controllable by the
production manager.
Answer (C) is incorrect because
flexible budgeting prepares budgets for Answer (C) is incorrect because
multiple levels of operations. repairs and maintenance are
controllable by the production manager.
Answer (D) is correct. Responsibility
Answer (D) is correct. A well-designed that provide for feedback reports
responsibility accounting system indicating deviations from expectations.
establishes responsibility centers within Higher-level management may focus on
an organization. In a responsibility those deviations for either
accounting system, managerial reinforcement or correction.
performance should be evaluated only
on the basis of those factors Answer (C) is incorrect because
controllable (or at least capable of authority is an element of a
being significantly influenced) by the responsibility accounting system, not
manager. Thus, a manager of an the basic purpose.
assembly line is responsible for direct
labor, materials, repairs and Answer (D) is incorrect because
maintenance, and supervisory salaries. analysis of variances is an element of a
The manager is not responsible for responsibility accounting system, not
depreciation on the manufacturing the basic purpose.
facility. (S)he is not in a position to
control or influence capital budgeting
decisions. [76] Source: CMA 0693 3-29

Answer (A) is incorrect because


[74] Source: CMA 0692 3-23 allocating depreciation on the basis of
long-term average use is a reasonable
Answer (A) is incorrect because basis of allocation. This basis is
nothing indicates that a flexible budget controllable by the division managers
is used. and reflects a causal relationship.

Answer (B) is correct. Responsibility Answer (B) is correct. Managerial


accounting stresses that managers performance ordinarily should be
should only be held responsible for evaluated only on the basis of those
factors under their control. To achieve factors controllable by the manager. If a
this objective, the operations of the manager is allocated costs that (s)he
business are broken down into cannot control, dysfunctional motivation
responsibility centers. Costs are can result. In the case of allocations, a
classified as controllable and cause-and-effect basis should be used.
noncontrollable to assign responsibility. Allocating the costs of upkeep on a
The assignment of responsibility headquarters building on the basis of
implies that some revenues and costs sales revenue is arbitrary because cost
can be changed through effective may have no relationship to divisional
management. A responsibility sales revenues. Consequently,
accounting system should have certain divisional ROI is reduced by a cost
controls that provide for feedback over which a division manager has no
reports indicating deviations from control. Furthermore, the divisions with
expectations. Management may then the greatest sales are penalized by
focus on those deviations for either receiving the greatest allocation.
reinforcement or correction.
Answer (C) is incorrect because a
Answer (C) is incorrect because the service department's cost overruns may
cost-benefit constraint on accounting not be attributable to any activities of
information is pervasive. The benefits production departments.
of the information provided by any
system should at least equal the cost. Answer (D) is incorrect because
market-based allocations of costs of
Answer (D) is incorrect because services are reasonable applications of
program budgeting is a system in which the cause-and-effect principle.
budgets are prepared by program rather
than by line item costs (such as salaries
or supplies). It is a long-term method [77] Source: CMA 0686 4-14
that associates outlays with the broad
objectives of the organization. Answer (A) is incorrect because sales
of the division would appear on the
statement.
[75] Source: CMA 0691 3-28
Answer (B) is incorrect because the
Answer (A) is incorrect because division's fixed selling expenses are
budgeting is an element of a separable fixed costs.
responsibility accounting system, not
the basic purpose. Answer (C) is incorrect because
variable costs of the division are
Answer (B) is correct. The basic
purpose of a responsibility accounting included.
system is to motivate management to
perform in a manner consistent with Answer (D) is correct. As defined in
overall company objectives. The Statement on Management Accounting
assignment of responsibility implies 2A, Management Accounting Glossary,
that some revenues and costs can be segment margin is the contribution
changed through effective management. margin for a segment of a business
The system should have certain controls minus fixed costs. It is a measure of
long-run profitability. Thus, an are classified as controllable and
allocation of the corporate officers' noncontrollable to assign responsibility,
salaries should not be included in which implies that some revenues and
segment margin because they are neither costs can be changed through effective
variable costs nor fixed costs that can management. For example, depreciation
be rationally allocated to the segment. on equipment is ordinarily not
Other items that are often not allocated controllable by the manager of an
include corporate income taxes, assembly line and should not appear on
interest, company-wide R&D expenses, his/her performance report.
and central administration costs.

[80] Source: CMA 1292 3-22


[78] Source: CIA 0587 IV-15
Answer (A) is incorrect because
Answer (A) is correct. Managerial unallocated fixed costs do not affect
performance should be evaluated only either performance measure.
on the basis of those factors
controllable by the manager. Managers Answer (B) is incorrect because direct
may control revenues, costs, and variable costs affect both performance
investment in resources. A measures.
well-designed responsibility accounting
system establishes responsibility Answer (C) is incorrect because costs
centers within the organization. The controllable by the manager affect both
sales department should therefore be performance measures.
responsible for the overtime costs
because it can best judge whether the
additional cost of the rush order is Answer (D) is correct. Control of costs
justified. Therefore, the sales accounts for the major difference
department may be charged with the between segment manager performance
overtime costs because it can best judge and segment performance. Segment
whether the additional cost of the rush performance is based on all costs
order is justified. The production directly attributable to the segment.
department may also be held Segment manager performance is based
responsible for the overtime costs on all costs directly controllable by the
because charging the full overtime cost segment manager. All variable costs
to the sales department would give the ordinarily meet the criteria for both
production department no incentive to measures. The difference usually arises
control these costs. However, the because a fixed cost is directly
personnel department would never be attributable to a segment but is not
charged with the overtime costs controllable by the manager. For
because it has no effect on the example, a profit center manager may
incurrence of production overtime. have no control over fixed costs of the
segment.
Answer (B) is incorrect because, to
control costs the production department
may be charged with the overtime costs. [81] Source: CMA 0691 3-29

Answer (C) is incorrect because, to Answer (A) is incorrect because total


control costs, the sales department may assets available does not reflect
be charged with the overtime costs. financing decisions.

Answer (D) is incorrect because, to Answer (B) is incorrect because total


control costs, the sales department and assets employed does not reflect
the production department may be financing decisions.
charged with the overtime costs.
Answer (C) is incorrect because
deducting current liabilities from
[79] Source: CMA 0691 3-26 current assets to arrive at working
capital reflects the control that the
Answer (A) is incorrect because manager of the responsibility center
supervisory salaries are costs ordinarily has over short-term credit
controllable by an assembly line transactions.
manager.
Answer (D) is correct. Stockholders'
Answer (B) is incorrect because equity equals total assets minus total
materials are costs controllable by an liabilities. The latter include short-term
assembly line manager. liabilities incurred at operating levels
of the organization and long-term
Answer (C) is incorrect because liabilities resulting from financing
repairs and maintenance are costs decisions made by top management.
controllable by an assembly line Accordingly, the investment base used
manager. to measure the performance of a
manager may reflect the incurrence of
Answer (D) is correct. Responsibility liabilities over which (s)he had no
accounting stresses that managers control. A second problem is that the
should be held responsible for only allocation of long-term liabilities
those factors under their control. Costs among divisions or segments may be
somewhat arbitrary.
Answer (B) is incorrect because
Sanders and Carolina would accept the
[82] Source: CMA 1291 3-7 project.

Answer (A) is incorrect because fixed Answer (C) is correct. Residual income
operating assets are controlled by the is the excess of the return on an
division manager and contribute to investment over a targeted amount,
profits. which is equal to an imputed interest
charge on invested capital (in this case,
Answer (B) is correct. An evaluation of 8%). The rate is usually the
an investment center is based upon the weighted-average cost of capital. Some
return on the investment base. These enterprises prefer to measure
assets include plant and equipment, managerial performance in terms of the
inventories, and receivables. Most amount of residual income rather than
likely, however, an asset, such as land, the percentage ROI. The principle is
that is being held by the division as a that the enterprise is expected to benefit
site for a new plant would not be from expansion as long as residual
included in the investment base because income is earned. Using a percentage
it is not currently being used in ROI approach, expansion might be
operations. Total assets in use rather rejected if it lowered ROI, even though
than total assets available is preferable residual income would increase. Using
when the investment center has been residual income, both Carolina and
forced to carry idle assets. Sanders would accept the new project
because residual income will increase
Answer (C) is incorrect because if a 12% return is earned when the
inventories are operating assets that target ROI is only 8%.
contribute to profits and are controlled
by the division manager. Answer (D) is incorrect because
Sanders and Carolina would accept the
Answer (D) is incorrect because the project.
level of accounts payable is an
operating decision that should be
considered in the evaluation of the [85] Source: CMA 0693 3-11
division manager.
Answer (A) is correct. A company with
an 8% ROI threshold should obviously
[83] Source: CMA 1292 3-21 accept a project yielding 12% because
the company's overall ROI would
Answer (A) is incorrect because increase. The manager being evaluated
increasing sales and expenses by the on the basis of ROI who is already
same dollar amount will not change earning 14% will be unwilling to
income or ROI. accept a 12% return on a new project
because the overall ROI for the division
Answer (B) is incorrect because would decline slightly. This absence of
decreasing revenues and expenses by goal congruence suggests a weakness in
the same percentage will reduce income ROI-based performance evaluation.
and lower ROI.
Answer (B) is incorrect because
Answer (C) is incorrect because Carolina would accept a project
increasing investment and operating yielding a return greater than 8%, and
expenses by the same dollar amount Sanders would reject a return yielding
will lower ROI. The higher investment less than 14%.
increases the denominator, and the
increased expenses reduce the Answer (C) is incorrect because
numerator. Carolina would accept a project
yielding a return greater than 8%, and
Answer (D) is correct. ROI equals Sanders would reject a return yielding
income divided by invested capital. If a less than 14%.
company is already profitable,
increasing sales and expenses by the Answer (D) is incorrect because
same percentage will increase ROI. For Carolina would accept a project
example, if a company has sales of yielding a return greater than 8%, and
$100 and expenses of $80, its net Sanders would reject a return yielding
income is $20. Given invested capital less than 14%.
of $100, ROI is 20% ($20 ÷ $100). If
sales and expenses both increase 10%
to $110 and $88, respectively, net [86] Source: CMA 0693 3-27
income increases to $22. ROI will then
be 22% ($22 ÷ $100). Answer (A) is incorrect because ROI
can be misleading when the quality of
the investment base differs among
[84] Source: CMA 0693 3-12 segments.

Answer (A) is incorrect because Answer (B) is correct. Return on


Sanders and Carolina would accept the investment is the key performance
project. measure in an investment center. ROI is
a rate computed by dividing a segment's equity, total assets available, or total
income by the invested capital. ROI is assets employed (which excludes assets
therefore subject to the numerous that are idle). Total assets available is
possible manipulations of the income the measure that assumes the manager
and investment amounts. For example, a will use all assets without regard to
manager may choose not to invest in a financing.
project that will yield less than the
desired rate of return, or (s)he may
defer necessary expenses. [89] Source: CMA 0695 3-20

Answer (C) is incorrect because Answer (A) is incorrect because,


managers may reject projects that are although the firm's return on equity
profitable (a return greater than the cost investment was 4%, its return on all
of capital), but would decrease ROI. funds invested was 5% ($25,000 pretax
For example, a segment with a 15% operating income ÷ $500,000).
ROI may not want to invest in a new
project with a 10% ROI, even though Answer (B) is correct. Return on
the cost of capital might be only 8%. investment is commonly calculated by
dividing pretax income by total assets
Answer (D) is incorrect because the use available. Residual income is the
of ROI does not reflect the relative excess of the return on investment over
difficulty of tasks undertaken by a targeted amount equal to an imputed
managers. interest charge on invested capital. The
rate used is ordinarily the
weighted-average cost of capital. Some
[87] Source: CMA 0694 3-18 companies measure managerial
performance in terms of the amount of
Answer (A) is incorrect because the residual income rather than the
cost of equity capital must also be percentage return on investment.
incorporated into the imputed interest Because REB has assets of $500,000
rate. and a cost of capital of 6%, it must earn
$30,000 on those assets to cover the
Answer (B) is incorrect because the cost of capital. Given that operating
current weighted-average cost of income was only $25,000, it had a
capital must be used. negative residual income of $5,000.

Answer (C) is correct. Residual income Answer (C) is incorrect because ROI is
is the excess of the return on an commonly based on before-tax income.
investment over a targeted amount equal
to an imputed interest charge on Answer (D) is incorrect because
invested capital. The rate used is $(22,000) equals the difference
ordinarily set as a target return by between net profit after taxes and
management but is often equal to the targeted income.
weighted average cost of capital. Some
enterprises prefer to measure
managerial performance in terms of the [90] Source: CMA 1296 3-2
amount of residual income rather than
the percentage ROI because the firm Answer (A) is incorrect because these
will benefit from expansion as long as ROI computations do not subtract
residual income is earned. imputed interest on capital used from
the investment base.
Answer (D) is incorrect because the
rate should be based on cost of capital, Answer (B) is correct. Residual income
not investment returns of preceding is the excess of the amount of return on
years. investment (ROI) over a targeted
amount equal to an imputed interest
charge on invested capital. The rate
[88] Source: CMA 0694 3-29 used to impute the interest is usually the
weighted-average cost of capital. The
Answer (A) is incorrect because ROI is advantage of using residual income
based on all assets, not just current rather than percentage ROI is that the
investment expenditures. former emphasizes maximizing an
amount instead of a percentage.
Answer (B) is incorrect because the Managers are encouraged to accept
calculation of ROI does not adjust for projects with returns exceeding the cost
imputed interest on invested capital. of capital even if the investments reduce
the percentage ROI.
Answer (C) is incorrect because the
denominator would not be limited to Answer (C) is incorrect because
fixed assets. operating income equals operating
revenues minus operating costs.
Answer (D) is correct. ROI is
calculated by dividing income by Answer (D) is incorrect because these
invested capital. It is a key performance ROI computations do not subtract
measure of an investment center. imputed interest on capital used from
Invested capital may be defined in the investment base.
various ways, such as shareholders'
achieve its goals while functioning in
[91] Source: CMA 1296 3-27 the best interest of the overall company.
Transfer prices can be determined in a
Answer (A) is incorrect because they number of ways, including normal
reflect an assumption that the subunit market price, negotiated price, variable
manager does not influence the resource costs, or full absorption costs. The
base (denominator of the ROI capacity of the Selling Division is often
calculation). a determinant of the ideal transfer price.
If the Fabricating Division had no
Answer (B) is correct. ROI equals excess capacity, it would charge the
income divided by invested capital. The Assembling Division the regular market
denominator may be defined in various price. However, if the Fabricating
ways, e.g., total assets available, assets Division has excess capacity of 1,000
employed, working capital plus other units, negotiation is possible because
assets, and shareholders' equity. If any transfer price greater than the
shareholders' equity (total assets - total variable cost of $20 would absorb
liabilities) is chosen, a portion of some of its fixed costs and result in
long-term liabilities must be allocated increased divisional profits. Thus, any
to the investment center to determine the price between $20 and $50 is
manager's resource base. One problem acceptable to the Fabricating Division.
with this definition of the resource base Any price under $50 is acceptable to
is that, although it has the advantage of the Assembling Division because that is
emphasizing return to owners, it reflects the price that would be paid to an
decisions at different levels of the outside supplier.
entity: short-term liabilities incurred by
the responsibility center (operating Answer (B) is incorrect because the
decisions) and long-term liabilities Assembling Division would not pay
controlled at the corporate level more than the market price of $50.
(long-term financing decisions).
Answer (C) is incorrect because
Answer (C) is incorrect because Fabricating will not be willing to
working capital plus other assets accept less than its variable cost of $20.
reflects the assumption that the manager
controls short-term credit. However, no Answer (D) is incorrect because
corporate-level decision to allocate Fabricating should be willing to accept
long-term liabilities to subunits is any price between $20 and $50.
necessary.

Answer (D) is incorrect because they [94] Source: CMA 0696 3-26
reflect an assumption that the subunit
manager does not influence the resource Answer (A) is incorrect because
base (denominator of the ROI evaluating the seller is difficult if it can
calculation). pass along all costs to the buyer.

Answer (B) is incorrect because


[92] Source: CMA 0692 3-14 transfers at full cost do not allow for a
seller's profit.
Answer (A) is incorrect because a
production center may be a cost center, Answer (C) is correct. A transfer price
a profit center, or even an investment is the amount one segment of an
center. Transfer prices are not used in a organization charges another segment
cost center. Transfer prices are used to for a product. The selling division
compute profitability, but a cost center should be allowed to recover its
is responsible only for cost control. incremental cost plus the opportunity
cost of the transfer. Hence, in a
Answer (B) is incorrect because an competitive market, the seller should be
investment center is not as fundamental able to charge the market price. Using
as a profit center. full cost as a transfer price provides no
incentive to the seller to control
Answer (C) is incorrect because production costs.
transfer prices are not used in a cost
center. Answer (D) is incorrect because a
full-cost transfer is favorable to the
Answer (D) is correct. Transfer prices buyer. It is lower than the market price.
are often used by profit centers and
investment centers. Profit centers are
the more fundamental of these two [95] Source: CMA 0696 3-27
centers because investment centers are
responsible not only for revenues and Answer (A) is incorrect because this
costs but also for invested capital. action is congruent with the goals of
Parkside. The use of idle capacity
enhances profits.
[93] Source: CMA 0694 3-30
Answer (B) is incorrect because the
Answer (A) is correct. An ideal transfer is at a loss (relative to full
transfer price should permit each cost) to the seller, although the company
division to operate independently and as a whole will benefit.
determining the transfer price that meets
Answer (C) is incorrect because the these criteria in all situations, a starting
buyer is indifferent as to whether to point is to calculate the sum of the
purchase internally or externally. additional outlay costs and the
opportunity cost to the supplier. Given
Answer (D) is correct. If the seller has no idle capacity and a competitive
excess capacity, it should lower its external market (all goods transferred
transfer price to match the outside offer. internally can be sold externally), the
This decision optimizes the profits of sum of the outlay and opportunity costs
the company as a whole by allowing for will be the market price.
use of capacity that would otherwise be
idle.
[98] Source: CIA 1188 IV-23

[96] Source: CMA 0696 3-28 Answer (A) is incorrect because market
price is an approach to determine a
Answer (A) is incorrect because this transfer price.
arrangement creates no disincentive for
the seller. It will make a profit on every Answer (B) is correct. A transfer price
unit transferred. is the price charged by one segment of
an organization for a product or service
Answer (B) is correct. Given that the supplied to another segment of the same
Plastics Division (the seller) has excess organization.
capacity, transfers within the company
entail no opportunity cost. Accordingly, Answer (C) is incorrect because outlay
the transfer at the negotiated price will price is an approach to determine a
improve the performance measures of transfer price.
the transferor. Purchasing internally at
below the market price also benefits the Answer (D) is incorrect because
transferee, so the motivational purpose distress price is an approach to
of transfer pricing is achieved. The goal determine a transfer price.
congruence purpose is also achieved
because the internal transaction benefits
the company. [99] Source: Publisher

Answer (C) is incorrect because the Answer (A) is correct. Motivation is


market price charged by outside sources the desire to attain a specific goal (goal
is higher than the negotiated price. congruence) and the commitment to
accomplish the goal (managerial effort).
Answer (D) is incorrect because, given Managerial motivation is therefore a
idle capacity, selling at any amount in combination of managerial effort and
excess of variable cost should motivate goal congruence.
the seller.
Answer (B) is incorrect because goal
congruence is the sharing of goals by
[97] Source: CMA 1296 3-17 supervisors and subordinates.

Answer (A) is incorrect because using Answer (C) is incorrect because


flexible budget cost as a transfer price autonomy is the extent to which
provides no motivation to the seller to individuals have the authority to make
control costs and no reward for selling decisions.
internally when an external market
exists. Answer (D) is incorrect because
managerial effort is the extent of the
Answer (B) is incorrect because using attempt to accomplish a specific goal.
incremental cost as a transfer price
provides no motivation to the seller to
control costs and no reward for selling [100] Source: Publisher
internally when an external market
exists. Answer (A) is incorrect because the
retail price is the definition of the
Answer (C) is incorrect because market market price, assuming an arm's-length
price is preferable to a budgeted or transaction.
actual cost with or without a markup
(unless the markup equals the profit Answer (B) is correct. At this price, the
earned by selling externally). supplying division is indifferent as to
whether it sells internally or externally.
Answer (D) is correct. Transfer prices Outlay cost plus opportunity cost
are the amounts charged by one segment therefore represents a minimum
of an organization for goods and acceptable price for a seller. However,
services it provides to another segment no transfer price formula is appropriate
within the organization. Transfer prices in all circumstances.

should promote congruence of subunit Answer (C) is incorrect because full


goals with those of the organization, cost is the price usually set by an
subunit autonomy, and managerial absorption-costing calculation.
effort. Although no rule exists for
Answer (D) is incorrect because the waste and maximize efficiency in a
variable-cost-plus price is the price set competitive economy (an outside
by charging for variable costs plus a market in which all padding produced
lump sum or an additional markup, but can be sold). This price also measures
less than full markup. the product's profitability and the
division managers' performance in a
competitive environment.
[101] Source: Publisher

Answer (A) is incorrect because the [103] Source: CIA 1190 IV-20
price on the open market is the
definition of the market price. Answer (A) is incorrect because
inefficiencies are charged to the buying
Answer (B) is incorrect because outlay department.
cost plus opportunity cost is the price
representing the cash outflows of the Answer (B) is correct. The optimal
supplying division plus the contribution transfer price of a selling division
to the supplying division from an should be set at a point that will have
outside sale. the most desirable economic effect on
the firm as a whole while at the same
time continuing to motivate the
Answer (C) is incorrect because the management of every division to
full-cost price is the price usually set by perform efficiently. Setting the transfer
an absorption-costing calculation. price based on actual costs rather than
standard costs would give the selling
Answer (D) is correct. The division little incentive to control costs.
variable-cost-plus price is the price set
by charging for variable cost plus either Answer (C) is incorrect because, by
a lump sum or an additional markup but definition, cost-based transfer prices
less than the full markup price. This are not adjusted by some markup.
permits top management to enter the
decision process and dictate that a Answer (D) is incorrect because
division transfer at variable cost plus cost-based transfer prices provide the
some appropriate amount. advantages of clarity and administrative
convenience.

[102] Source: CIA 1191 IV-19


[104] Source: CIA 0595 III-96
Answer (A) is incorrect because the
market price will better achieve the Answer (A) is incorrect because return
goals of a transfer pricing system. The on assets cannot be computed for a cost
selling unit would not have as strong an center. The manager is not responsible
incentive to control costs if some for revenue (return) or the assets
variant of actual cost is used. The available.
efficiency of the purchasing unit is also
promoted when it must treat the selling Answer (B) is incorrect because return
unit as if it were an independent vendor. on investment cannot be computed for a
cost center. The manager is not
Answer (B) is incorrect because the responsible for revenue (return) or the
market price will better achieve the assets available.
goals of a transfer pricing system. The
selling unit would not have as strong an Answer (C) is incorrect because the
incentive to control costs if some payback method is a means of
variant of actual cost is used. evaluating alternative investment
proposals.
Answer (C) is incorrect because the
market price will better achieve the Answer (D) is correct. A cost center is
goals of a transfer pricing system. The a responsibility center that is
selling unit would not have as strong an responsible for costs only. Of the
incentive to control costs if some alternatives given, variance analysis is
variant of actual cost is used. the only one that can be used in a cost
center. Variance analysis involves
Answer (D) is correct. The three basic comparing actual costs with predicted
criteria that the transfer pricing system or standard costs.
in a decentralized company should
satisfy are to: (1) provide information
allowing central management to [105] Source: CMA 0697 3-29
evaluate divisions with respect to total
company profit and each division's Answer (A) is incorrect because
contribution to profit, (2) stimulate each 34.78% results from subtracting
manager's efficiency without losing working capital from plant and
each division's autonomy, and (3) equipment in calculating the net
motivate each divisional manager to investment.
achieve his/her own profit goal in a
manner contributing to the company's Answer (B) is incorrect because
success. The market price should be 22.54% fails to include average
used as the transfer price to avoid working capital in the total for the net
investment. Answer (D) is incorrect because
Scenario 4 requires maximum costs of
Answer (C) is incorrect because $25,600,000 to reach the target.
19.79% results from not subtracting
general and administrative expenses in
the calculation of before-tax profit. [108] Source: CMA 0691 3-26

Answer (D) is correct. An investment Answer (A) is incorrect because


center is responsible for revenues, supervisory salaries are costs
expenses, and invested capital. Given controllable by an assembly line
average plant and equipment of $1,775 manager.
and average working capital of $625,
the net investment is $2,400. Before-tax Answer (B) is incorrect because
profit is $400 ($4,000 sales - $3,525 materials are costs controllable by an
cost of goods sold - $75 general assembly line manager.
expenses). If before-tax ROI equals
before-tax profit divided by net Answer (C) is incorrect because
investment, the answer is 16.67% ($400 repairs and maintenance are costs
÷ $2,400). controllable by an assembly line
manager.

[106] Source: CMA 0697 3-24 Answer (D) is correct. Responsibility


accounting stresses that managers
Answer (A) is incorrect because the should be held responsible for only
methods use the same asset base. those factors under their control. Costs
are classified as controllable and
Answer (B) is correct. Residual income
is the excess of the amount of the ROI noncontrollable to assign responsibility,
over a targeted amount equal to an which implies that some revenues and
imputed interest charge on invested costs can be changed through effective
capital. The advantage of using residual management. For example, depreciation
income rather than percentage ROI is on equipment is ordinarily not
that the former emphasizes maximizing controllable by the manager of an
a dollar amount instead of a percentage. assembly line and should not appear on
Managers of divisions with a high ROI his/her performance report.
are encouraged to accept projects with
returns exceeding the cost of capital
even if those projects reduce the [109] Source: Publisher
department's ROI.
Answer (A) is incorrect because the
Answer (C) is incorrect because the return on sales was 3.33%.
methods use the same asset base.
Answer (B) is correct. Residual income
Answer (D) is incorrect because use of is the excess of the actual ROI in
the residual income method requires a dollars over a targeted amount equal to
knowledge of the cost of capital; thus, an imputed interest charge on invested
arguments about the implicit cost of capital. The rate used is ordinarily the
interest may escalate with use of the weighted-average cost of capital. Some
residual income method. entities measure managerial
performance in terms of the amount of
residual income rather than the
[107] Source: Publisher percentage ROI. Assuming the
investment base is defined as total
Answer (A) is correct. Residual income assets available, Charlie's targeted
is the excess of the amount of the ROI amount is $30,000 ($500,000 total
over a targeted amount equal to an assets x 6% cost of capital). Assuming
imputed interest charge on invested that operating income of $25,000 is the
capital. If a manager has $19,000,000 ROI in dollars, residual income was
of invested capital ($17,200,000 of $(5,000). This result is consistent with
plant and equipment + $1,800,000 of defining the numerator of the ROI
working capital), a 15% imputed calculation (Income ÷ Investment) as
interest charge equals $2,850,000. operating income. However, it might
Adding $2,000,000 of residual income also be defined as net profit after taxes
to the imputed interest results in a target (net income). Moreover, the ROI
profit of $4,850,000. This profit can be denominator may be defined variously,
achieved if costs are $25,150,000 e.g., total assets available, total assets
($30,000,000 revenue - $4,850,000 employed, working capital plus other
profit). assets, or shareholders' equity.

Answer (B) is incorrect because Answer (C) is incorrect because 6% is


Scenario 2 requires maximum costs of the cost of capital.
$26,220,000 to reach the target.
Answer (D) is incorrect because
Answer (C) is incorrect because $(20,000) assumes that ROI in dollars
Scenario 3 requires maximum costs of is $25,000 (operating income) and that
$25,330,000 to reach the target. the targeted amount is $45,000 (6% x
$750,000 of sales).
Answer (D) is incorrect because
$1,680,000 is the after-tax operating
[110] Source: CMA Samp Q3-2 income and does not deduct the cost of
capital.
Answer (A) is correct. The CM equals
revenues minus all variable costs
expensed. Given no WIP and no [112] Source: Publisher
beginning finished goods, the CM was
$25,200 [($100 - $30 - $20 - $10 - Answer (A) is incorrect because 8% is
$12) x 900 units]. The variable costs of the pre-tax cost of debt.
producing the units not sold are
included in ending inventory, rather than Answer (B) is incorrect because 8.89%
in the CM. The fixed costs are also is based on the book value of equity.
excluded from computation of the CM.
Answer (C) is correct. The WACC is
Answer (B) is incorrect because an after-tax rate determined using the
$28,000 results from assuming the sale fair values of the sources of long-term
of 1,000 units. funds. Thus, the appropriate cost of debt
is 6% [.08 x (1.0 - .25 tax rate)]
Answer (C) is incorrect because because interest is tax deductible.
$31,500 results from assuming a UCM However, the given equity rate (10%)
of $35. This computation includes fixed is not adjusted because distributions to
unit selling costs of $5 but excludes the shareholders are not deductible. The
$12 per unit variable selling costs. fair value of long-term debt is given as
$5 million. The book value of equity
Answer (D) is incorrect because must be $13 million ($20 million of
$35,000 results from assuming a UCM assets - $7 million of liabilities), and
of $35 and sales of 1,000 units. its fair value is $15 million ($13
million + $2 million). Accordingly, the
WACC is 9%:
[111] Source: Publisher
(.06 x $5 million FV of LT debt) +
Answer (A) is incorrect because (10% x $15 million FV of equity)
$731,240 is the cost of capital. WACC = ??????????????????????????????????
($5 million + $15 million)
Answer (B) is correct. This problem $300,000 + $1.5 million
involves several steps. EVA equals = ???????????????????????
after-tax operating income minus the $20 million
product of the after-tax WACC and an = .09
investment base equal to total assets
minus current liabilities. After-tax Answer (D) is incorrect because 10%
operating income and WACC are not is the cost of equity.
given in the problem and have to be
calculated. WACC is an after-tax rate
determined using the fair values of the [113] Source: Publisher
sources of long-term funds. Interest is
tax deductible, so the coupon rate on Answer (A) is correct. EVA equals
long-term debt has to be adjusted as after-tax operating income minus the
such, so that the appropriate cost of product of the after-tax WACC and an
debt for WACC calculations is 4.2% investment base equal to total assets
[0.06(1 - 0.3 tax rate)]. The given minus current liabilities. Thus, EVA is
equity rate of 8% is not altered because $1,380,000 {[$4 million x (1.0 - .25)] -
distributions to shareholders are not [.09 after-tax WACC x ($20 million
deductible. The fair value of long-term total assets - $2 million current
debt is given as $2,200,000. The book liabilities)]}.
value of equity has to be total assets
minus total liabilities, or $7,900,000. Answer (B) is incorrect because
Therefore, its fair value is $900,000 $1,620,000 is the required return on the
more than its book value, or investment base.
$8,800,000. The WACC is then
calculated as follows: Answer (C) is incorrect because
$1,830,000 is based on the assumption
[(.042)($2,200,000 FV of LT debt)] that $13 million is the investment base.
+ [(0.08)($8,800,000 FV of equity)]
WACC = ??????????????????????????????????? = Answer (D) is incorrect because
7.24% $3,000,000 is the after-tax operating
($2,200,000 + $8,800,000 income.
After-tax operating income is operating
income multiplied by 1 minus the tax
rate, which equals $1,680,000. Thus, [114] Source: CMA 0697 3-30
EVA is equal to ($1,680,000) -
[(0.0724)(10,100,000)], or $948,760. Answer (A) is incorrect because this
level of cost would result in a residual
Answer (C) is incorrect because income greater than $2,000,000.
income taxes must be deducted from
operating income to compute EVA. Answer (B) is incorrect because this
level of cost would result in a residual
income greater than $2,000,000. Answer (D) is incorrect because the
accrual return on investment in the first
Answer (C) is correct. Residual income years of a capital project may be very
is the excess of the amount of the ROI low even though it has a positive net
over a targeted amount equal to an present value.
imputed interest charge on invested
capital. If a manager has $19,000,000
of invested capital ($17,200,000 of [117] Source: CIA 1195 II-36
plant and equipment + $1,800,000 of
working capital), a 15% imputed Answer (A) is incorrect because
interest charge equals $2,850,000. employees will become discouraged if
Adding $2,000,000 of residual income the goals cannot be met.
to the imputed interest results in a target
profit of $4,850,000. This profit can be Answer (B) is incorrect because goals
achieved if costs are $25,150,000 should be quantitative and specific.
($30,000,000 revenue - $4,850,000 They should not be too abstract.
profit).
Answer (C) is incorrect because not all
Answer (D) is incorrect because employees are superior. Employees are
$25,690,000 results from subtracting discouraged by unreachable goals.
working capital from plant and Goals should be based on an
equipment in determining invested employee's skills and capacity for
capital. improvement.

Answer (D) is correct. Effective goal


[115] Source: Publisher setting requires a sufficient knowledge
of employees' jobs to set specific,
Answer (A) is incorrect because a objective, verifiable goals. Employees
manager should be motivated to expend must also understand how goal-oriented
the effort required to achieve performance will be measured.
organizational objectives.

Answer (B) is incorrect because a [118] Source: Publisher


manager should be motivated to expend
the effort required to achieve Answer (A) is incorrect because
organizational objectives. authority is the power to direct and
exact performance from others. It
Answer (C) is correct. A managerial includes the right to prescribe the means
control system should encourage the and methods by which work will be
done.
efficient achievement of organizational
objectives. Thus, goal congruence and Answer (B) is incorrect because
managerial effort are aspects of responsibility is the obligation to
motivation. All managers should be perform.
motivated to expend the necessary effort
to reach common goals. Answer (C) is incorrect because
accountability is the liability for failure
Answer (D) is incorrect because a to meet the obligation.
manager should be motivated to expend
the effort required to achieve Answer (D) is correct. Controllability
organizational objectives. is the extent to which a manager can
influence activities and related
revenues, costs, or other items. In
[116] Source: Publisher principle, controllability is
proportionate to, but not coextensive
Answer (A) is incorrect because a with, responsibility.
cost-based transfer price may provide
no incentive to minimize cost.
[119] Source: CMA 0693 3-21
Answer (B) is correct. If the measure of
the retail store manager's performance Answer (A) is incorrect because, if the
does not consider opportunity costs, for costs are not allocated to individual
example, the cost of financing managers, they have no reason to object
inventory, (s)he may be tempted to to the allocation.
make suboptimal decisions about
inventory levels. Hence, ordering Answer (B) is correct. Control is the
excessive inventory may help to process of making certain that plans are
minimize the costs for which the achieving the desired objectives. A
manager is responsible while budget is one of the most common
increasing the costs for which (s)he is control devices. It is a plan for the
not. future; it is not a contract. To interpret a
budget or other plan to be as inflexible
Answer (C) is incorrect because the as a contract may encourage a manager
organization may be better served if a to act in ways contrary to the company's
manager maximizes an absolute dollar best interest in a misguided effort to
amount rather than a rate of return. meet the criteria proposed.
Answer (C) is incorrect because
participatory budgeting obtains the [122] Source: Publisher
support of those involved and is likely
to foster desirable behavior. Answer (A) is incorrect because they
are characteristics that Theory X
Answer (D) is incorrect because managers attribute to their employees.
changing budget targets as conditions
change results in setting fairer Answer (B) is incorrect because they
performance goals. are characteristics that Theory X
managers attribute to their employees.

[120] Source: Publisher Answer (C) is incorrect because they


are characteristics that Theory X
Answer (A) is incorrect because managers attribute to their employees.
disadvantages of participatory
budgeting and standard setting include Answer (D) is correct. MBO managers
the cost in terms of time and money. believe that employees are committed
to achieving objectives, working hard
Answer (B) is incorrect because the to receive the rewards of achievement,
quality of participation is affected by and striving for self-actualization. The
the goals, values, beliefs, and MBO view is that employees enjoy
expectations of those involved. work, need little supervision, seek
responsibility, and are imaginative
Answer (C) is incorrect because an problem solvers.
advantage of participatory budgeting is
that it yields information known to
employees but not to management. [123] Source: Publisher

Answer (D) is correct. Participatory Answer (A) is incorrect because


budgeting (grass-roots budgeting) and multiple measures are preferable.
standard setting use input from
lower-level and middle-level Answer (B) is incorrect because
employees. Participation encourages objectives should be specific.
employees to have a sense of ownership
of the output of the process. The result Answer (C) is correct. Budgets and
is an acceptance of, and commitment to, standards are often the result of a
the goals expressed in the budget. formal management-by-objectives
program. MBO is a top-down process
because the organization's objectives
[121] Source: CMA 0693 3-26 are successively restated into
objectives for each lower level.
Answer (A) is incorrect because However, it is also a bottom-up process
decentralizing decision-making because of the participation of
authority leads to more effective subordinates.
solutions to operational problems.
Solutions will come from the Answer (D) is incorrect because a
individuals with the greatest knowledge specific time period is established.
of the operations involved.

Answer (B) is incorrect because, when [124] Source: Publisher


segments are autonomous, other
segments are regarded as external Answer (A) is incorrect because MBO
parties, e.g., as suppliers, customers, or goals may be set in terms of quantitative
competitors. measures (such as sales dollars) or
qualitative ones (such as improved
Answer (C) is incorrect because service).
autonomous segments may have the
authority to compete in the same Answer (B) is incorrect because it
markets. describes management by exception.

Answer (D) is correct. Decentralization Answer (C) is correct. The hallmark of


is beneficial because it creates greater MBO is the mutual setting of goals by
responsiveness to the needs of local the superior and the subordinate as a
customers, suppliers, and employees. basis for performance evaluation.
Managers at lower levels are more Based on the Theory Y philosophy that
knowledgeable about local markets and employees want to work hard if they
the needs of customers, etc. A know what is expected, MBO requires
decentralized organization is also more top management participation and
likely to respond flexibly and quickly to commitment to the program, integration
changing conditions, for example, by of the objectives for all subunits into a
expediting the introduction of new compatible system directed toward
products. Furthermore, greater authority accomplishment of overall goals,
enhances managerial morale and provision for regular reporting of
development. Disadvantages of performance, and free and honest
decentralization include duplication of communication between superior and
effort and lack of goal congruence. subordinates. Subordinates must make
careful assessments of their abilities
and their interests, and managers must Answer (B) is incorrect because current
"coach" subordinates rather than dictate disposal price is an attempt to remedy
their proper goals. Both sides must the theoretical deficiencies of historical
maintain flexibility to accommodate cost by presenting more accurate
unforeseen changes, and the review and balance sheet values.
analysis of results before setting the
next round of goals is a vital part of the Answer (C) is correct. Historical cost
process. creates comparability issues because
returns on significantly depreciated
Answer (D) is incorrect because goal assets may be higher than those on
setting should be participative. newer assets that have been acquired
using inflated dollars. Thus, otherwise
similarly situated managers may report
[125] Source: Publisher different operating results. Moreover,
managers may be reluctant to replace
Answer (A) is incorrect because, when aging assets.
an employee believes that (s)he is in
control, (s)he feels more responsible Answer (D) is incorrect because
for achieving goals. present value is an attempt to remedy
the theoretical deficiencies of historical
Answer (B) is correct. MBO is the cost by presenting more accurate
process by which a manager and his/her balance sheet values.
subordinate(s) work together to
formulate the objectives and goals of
the subordinate(s). One of the [128] Source: Publisher
weaknesses of MBO is that emphasis on
quantitative factors may cause Answer (A) is incorrect because
employees to focus on ends rather than self-interest provides an incentive to
means. Thus, MBO may jeopardize the maximize the measures used in
quality of the organization's output. performance evaluation.

Answer (C) is incorrect because Answer (B) is correct. Effective


employee participation in goal setting management control requires
helps clarify goals and motivate the performance measurement and
employee. feedback. This process affects
allocation of resources to
Answer (D) is incorrect because an organizational subunits. It also affects
employee's trust in the team approach decisions about managers'
helps make the work atmosphere more compensation, advancement, and future
positive. assignments. Furthermore, evaluating
their performance serves to motivate
managers to optimize the measures in
[126] Source: Publisher the performance evaluation model.
However, that model may be
Answer (A) is incorrect because market inconsistent with the organization's
share is an external nonfinancial model for managerial decision making.
measure.
Answer (C) is incorrect because a
Answer (B) is incorrect because manager evaluated on the basis of
delivery performance is an external annual ROI has an interest in
nonfinancial measure. maximizing short-term net income, not
long-term NPV.
Answer (C) is incorrect because
customer satisfaction is an external Answer (D) is incorrect because the
nonfinancial measure. models should be synchronized so that
the goals of the organization and the
Answer (D) is correct. Feedback manager are congruent.
regarding managerial performance may
take the form of financial and
nonfinancial measures that may be [129] Source: Publisher
internally or externally generated.
Moreover, different measures have a Answer (A) is correct. A manager who
long-term or short-term emphasis. does not control an activity may
Examples of internal nonfinancial nevertheless be the individual who is
measures are product quality, new best informed about it. Thus, a
product development time, and purchasing agent may be in the best
manufacturing lead time (cycle time). position to explain price variances even
though (s)he cannot control them.
Moreover, if a manager is accountable
[127] Source: Publisher solely for activities over which (s)he
has extensive influence, the manager
Answer (A) is incorrect because may develop too narrow a focus. For
current cost is an attempt to remedy the example, the manager of a cost center
theoretical deficiencies of historical may make decisions based only on cost
cost by presenting more accurate efficiency and ignore the overall
balance sheet values. effectiveness goals of the organization.
By extending the manager's
responsibility to profits as well as costs Answer (D) is incorrect because the
without changing his/her level of benefits-received criterion is preferable
control, the organization may encourage when a cause-effect relationship cannot
desirable behavior congruent with be feasibly identified.
overall goals, such as improved
coordination with marketing personnel.
[132] Source: Publisher
Answer (B) is incorrect because the
potential for favorable behavioral Answer (A) is incorrect because an
change may justify assigning arbitrary allocation may skew operating
responsibility without control. results.

Answer (C) is incorrect because if Answer (B) is incorrect because the


responsibility exceeds the extent to allocation may create resentment and
which a manager can influence an conflict.
activity, the result may be reduced
morale, a decline in managerial effort, Answer (C) is correct. The allocation
and poor performance. reminds managers that support costs
exist and that the managers would incur
Answer (D) is incorrect because if these costs if their operations were
responsibility exceeds the extent to independent. The allocation also
which a manager can influence an reminds managers that profit center
activity, the result may be reduced earnings must cover some amount of
morale. support costs.

Answer (D) is incorrect because


[130] Source: Publisher efficient use of central support services
should be encouraged.
Answer (A) is incorrect because direct
costs can be traced to a particular cost
object in an economically feasible [133] Source: Publisher
manner.
Answer (A) is incorrect because a
Answer (B) is incorrect because current static (fixed) budget is based on one
cost is an attribute used to measure level of production and is not useful for
assets. other activity levels.

Answer (C) is incorrect because Answer (B) is incorrect because a


controllable costs can be influenced by static (fixed) budget is based on one
a particular manager. level of production and is not useful for
other activity levels.
Answer (D) is correct. Common costs
are the cost of products, activities, Answer (C) is incorrect because a
facilities, services, or operations continuous budget is revised (extended)
shared by two or more cost objects. on a regular basis. It may or may not be
They are indirect costs because they a flexible budget.
cannot be traced to a particular cost
object in an economically feasible Answer (D) is correct. A flexible
manner. Hence, they must be allocated. budget is actually a series of several
budgets prepared for many levels of
sales. At the end of the period,
[131] Source: Publisher management can compare actual costs
or performance with the appropriate
Answer (A) is correct. The difficulty budgeted level in the flexible budget.
with common costs is that they are Thus, flexible budgeting facilitates
indirect costs whose allocation may be variance analysis because it allows for
arbitrary. A direct cause-and-effect the effects of varying the activity level
relationship between a common cost so that price, efficiency, and other
and the actions of the cost object to variances can be isolated.
which it is allocated is desirable. Such
a relationship promotes acceptance of
the allocation by managers who [134] Source: Publisher
perceive the fairness of the procedure,
but identification of cause and effect Answer (A) is incorrect because the
may not be feasible. balanced scorecard approach uses
multiple measures.
Answer (B) is incorrect because
allocation using an ability-to-bear Answer (B) is correct. The trend in
criterion punishes successful managers managerial performance evaluation is
and rewards underachievers. the balanced scorecard approach.
Multiple measures of performance
Answer (C) is incorrect because permit a determination as to whether a
fairness is an objective rather than a manager is achieving certain objectives
criterion. Moreover, fairness may be at the expense of others that may be
interpreted differently by different equally or more important. These
managers. measures may be financial or
nonfinancial and usually include items
in four categories: profitability; are overriding, common goals of the
customer satisfaction; innovation; and parties. The conflict can be resolved if
efficiency, quality, and time. the individuals involved understand that
it is preventing them from achieving
Answer (C) is incorrect because the more important, mutually held goals.
balanced scorecard approach uses
financial and nonfinancial measures.
[138] Source: CIA 0594 II-9
Answer (D) is incorrect because the
balanced scorecard approach uses Answer (A) is incorrect because paying
financial and nonfinancial measures. a bonus is a positive reinforcement.

Answer (B) is correct. Negative


[135] Source: CIA 0594 III-50 reinforcement removes an unpleasant
condition when the desired behavior
Answer (A) is incorrect because teams occurs, whereas positive reinforcement
support team members. rewards the desired behavior. Thus,
attending class is reinforced by the
Answer (B) is incorrect because teams removal of something unpleasant, i.e.,
make better decisions than individuals. the receipt of a written warning.
Because a warning is given after every
Answer (C) is correct. In a culture that other absence, the reinforcement is
strongly emphasizes individual identity intermittent, not continuous.
and competition, the preference tends to
be for a clear link between effort and Answer (C) is incorrect because
outcome. However, teams tend to assigning a mentor is a positive
submerge individual identity and reinforcement. The firm is attempting to
responsibility and therefore to blur the link each individual with a positive role
link between individual effort and its model.
results.
Answer (D) is incorrect because
Answer (D) is incorrect because teams holding a lottery is an intermittent
control and discipline members. positive reinforcement.

[136] Source: CIA 1193 III-2 [139] Source: CIA 0594 III-89

Answer (A) is incorrect because, in an Answer (A) is incorrect because stating


organizational orientation session, the undesirable behavior clarifies for
employees receive formal information the employee the link between conduct
about objectives, the organizational and consequences.
chart, benefits, and procedures.
Answer (B) is correct. Effective
Answer (B) is correct. A manager's discipline requires immediate
knowledge and skills are broadened by corrective action to eliminate the
serving in a number of different negative effects of the undesirable
capacities. To a significant degree, employee conduct and to establish and
research shows that managers learn to reinforce appropriate behavior. Delay
manage by managing. merely invites more serious
consequences. Moreover, the
Answer (C) is incorrect because role punishment should be commensurate
playing is assuming different roles in with the offense, and the employee
different situations according to the should clearly perceive the relationship
expectations of the group. between the punishment and the
behavior.
Answer (D) is incorrect because liaison
committees facilitate understanding Answer (C) is incorrect because, in
between management and employees. U.S. legal culture, the accused has the
right to be heard in his/her defense.

[137] Source: CIA 0594 III-81 Answer (D) is incorrect because


focusing on the offense rather than the
Answer (A) is incorrect because offender is less likely to engender fear
conflict triggers include ambiguous and resentment on the part of the
jurisdictions (unclear job boundaries). employee.

Answer (B) is incorrect because


conflict triggers include competition for [140] Source: CIA 0594 III-83
scarce resources.
Answer (A) is incorrect because
Answer (C) is incorrect because management can change people's
conflict triggers include status abilities, attitudes, expectations, and
differentials. behavior.

Answer (D) is correct. An appeal to Answer (B) is incorrect because


superordinate goals is a means of management can change lines of
resolving conflict. Superordinate goals authority and responsibility, the nature
of jobs, and control mechanisms. Answer (B) is incorrect because the
limitations on rationality suggest that
Answer (C) is correct. The factors a individuals will seek limited
change agent may alter can be classified information that is readily available and
into three categories: structure, familiar to them, thereby reducing the
technology, and people. Organizational effectiveness of decision making.
change is most likely to be an
adaptation to change in the environment. Answer (C) is correct. Bounded
rationality is the concept that recognizes
Answer (D) is incorrect because the limitations on an individual's ability
management can change how work is to process information. Thus, presenting
performed. information in simpler forms assists the
decision maker. However, the danger of
constructing models that attempt to
[141] Source: CIA 1194 II-26 manage complexity by isolating the
essential features of a situation is that
Answer (A) is incorrect because important elements may be omitted.
superordinate goals are shared goals
that can be achieved only through Answer (D) is incorrect because time
cooperation. limitations only serve to reduce the
information available to the decision
Answer (B) is correct. Smoothing is a maker and lead to decisions that are
conflict resolution technique in which similar to previous decisions.
differences are deemphasized and
common interests of the parties are
emphasized. It has the disadvantage of [144] Source: CIA 0595 II-34
not solving the underlying problems that
created the conflict. Answer (A) is correct. Group decision
making is often characterized by greater
Answer (C) is incorrect because acceptance of risk because of the
problem solving involves identifying dispersal of accountability. Individual
and correcting the source of the conflict. decision making tends to be more
conservative because accountability can
Answer (D) is incorrect because be specifically assigned.
compromise requires each party to give
up something. Answer (B) is incorrect because a
group has greater resources of
knowledge and experience than an
[142] Source: CIA 0595 II-32 individual.

Answer (A) is correct. A company's Answer (C) is incorrect because a


mission statement defines crucial group has a wider diversity of views
organizational values and establishes a and should be able to offer a wider
basis for strategic planning. This range of solutions.
company's profit emphasis is an
example of a goals attainment approach Answer (D) is incorrect because a
to measuring performance. The disadvantage of group decision making
emphasis is on the results attained, not is that it is more time consuming than
the means used in pursuing the results. individual decision making.

Answer (B) is incorrect because the


goals emphasis approach is not best [145] Source: CIA 1195 II-1
suited for companies in the formation
stage. Companies in the formative Answer (A) is incorrect because
stages need to focus on obtaining scarce responsibility for group decisions is
resources and developing a strategy for more diffuse than when individuals
long-run survival. make decisions.

Answer (C) is incorrect because the Answer (B) is incorrect because group
company has not effectively decision making almost always takes
implemented the strategic constituencies more time than individual decision
approach. Only one of the organization's making, except when the need for
important constituencies is considered. diverse views is so great that an
individual decision maker needs to
Answer (D) is incorrect because the consult many people or perform
company has not effectively research.
implemented the strategic constituencies
approach. Only one of the organization's Answer (C) is incorrect because group
important constituencies is considered. members usually have diverse views,
but their common need to be accepted
and respected by the group often
[143] Source: CIA 0595 II-35 restrains the full, open expression of
their views when they fear strong
Answer (A) is incorrect because the disagreement.
most available information may not be
the best and most useful. Answer (D) is correct. Groups tend to
be more creative than individuals.
Diversity of member views, cheaper, but it may disrupt the
experiences, and abilities usually workplace and cause increased errors.
results in considering more solutions to Classroom lectures are appropriate
a problem. when the skill training is too complex to
be conducted on the job.

[146] Source: CIA 1195 II-15


[149] Source: CIA 1195 II-33
Answer (A) is incorrect because a
leniency bias would result in ratings Answer (A) is incorrect because a
that were consistently high for all quality circle is a small group of
employees. subordinates and supervisors, usually
eight to 10 people.
Answer (B) is incorrect because
leading questions would tend to Answer (B) is incorrect because each
produce similar ratings for different member is responsible for the success
individuals, assuming all subordinates of the circle, and success depends on
are rated using the same questions. the ability of members to analyze and
solve problems.
Answer (C) is correct. The halo effect
is a source of systematic bias. It causes Answer (C) is incorrect because quality
a judgment about one characteristic to circles are used by companies to
affect judgments about others. For accomplish objectives. Participation is
example, a perception about one trait of part of each worker's job.
a person may influence perceptions of
other qualities of that individual. Answer (D) is correct. Use of quality
circles is a form of participative
Answer (D) is incorrect because, if the management. A quality circle is a group
supervisor were trying to treat everyone of up to 10 individuals (managers and
the same, the variability among the subordinates) who do similar work and
individuals rated would be lower. who volunteer to meet weekly to
discuss and solve work-related
problems. However, management
[147] Source: CIA 1195 II-16 retains the right to make the final
decisions.
Answer (A) is incorrect because
authoritative command is a conflict
resolution technique. [150] Source: CIA 1195 II-36

Answer (B) is correct. Competition is a Answer (A) is incorrect because


conflict-handling intention employees will become discouraged if
characterized by considerable the goals cannot be met.
assertiveness (the degree to which the
party seeks to achieve his/her goals) Answer (B) is incorrect because goals
and a low degree of cooperativeness should be quantitative and specific.
(the degree to which the party attempts They should not be too abstract.
to satisfy the concerns of others).
Encouraging competition stimulates Answer (C) is incorrect because not all
conflict. employees are superior. Employees are
discouraged by establishing
Answer (C) is incorrect because unreachable goals. Goals should be
altering the structural variables is a based on an employee's skills and
conflict resolution technique. capacity for improvement.

Answer (D) is incorrect because Answer (D) is correct. Effective goal


compromise is a conflict resolution setting requires a sufficient knowledge
technique. of employees' jobs to set specific,
objective, verifiable goals. Employees
must also understand how goal-oriented
[148] Source: CIA 1195 II-18 performance will be measured.

Answer (A) is incorrect because


classroom instruction is usually less [151] Source: CIA 1194 II-28
expensive than on-the-job training.
Answer (A) is incorrect because
Answer (B) is incorrect because more classroom training permits the
material can be covered in less time employee to learn to use the actual
with the lecture method. job-related equipment in a training
setting.
Answer (C) is incorrect because any
level of depth can be achieved in the Answer (B) is incorrect because videos
lecture format. provide the specific, technical details
necessary to perform the job and can be
Answer (D) is correct. The principal viewed until the employee is
advantage of learning by experience is comfortable with the material.
that what is learned is remembered
much better. On-the-job training is also Answer (C) is incorrect because
apprenticeships place the employee group possesses greater resources than
with an experienced worker who serves an individual.
as a model for the understudy to
observe. Answer (D) is correct. The groupthink
phenomenon is undesirable. Groupthink
Answer (D) is correct. According to occurs when group members accept
Robbins (Organizational Behavior, pg. what appears to be the group consensus
565), simulation exercises, such as case rather than giving their honest input. The
analyses, role playing, experiential result may be decisions with which
exercises, and group interaction some members of the group are not
sessions, best serve in developing happy.
problem-solving and interpersonal
skills.
[155] Source: CIA 0596 II-20

[152] Source: CIA 1196 II-38 Answer (A) is incorrect because an


experienced employee or mentor from
Answer (A) is incorrect because values the same company might be a good
are stable and enduring. source of information, but someone
hired at the same time will not have
Answer (B) is incorrect because better information about how the
questioning values may result in their company operates.
reinforcement.
Answer (B) is incorrect because
Answer (C) is incorrect because values outsiders will not know details of how
are relatively fixed and change only the corporate culture operates.
slowly.
Answer (C) is correct. The corporate
Answer (D) is correct. Values are culture of an entity consists of the
specific to each individual and involve complex of shared values and
moral and personal issues. They tend to acceptable behaviors unique to the
be learned in childhood from parents, organization. Although some companies
friends, and others. Values can be formally create a corporate culture and
modified throughout life but ordinarily provide explicit orientation regarding
tend to stay the same. it, new employees in most companies
must learn what is appropriate by
observation and experience.
[153] Source: CIA 0596 II-33
Answer (D) is incorrect because, aside
Answer (A) is incorrect because from using valuable work time, this
personal beliefs alone are not an approach also obtains worthwhile
appropriate basis for managerial action. information only from managers who
truly understand the corporate culture.
Answer (B) is incorrect because Moreover, the information may be
personal beliefs alone are not an conflicting.
appropriate basis for managerial action.

Answer (C) is correct. The only [156] Source: CIA 0596 II-32
legitimate grounds on which the
supervisor may take action is the Answer (A) is incorrect because
employee's behavior. Personal beliefs, defining the task decreases both
such as those on religious and political acceptance and understanding of the
matters, cannot be the basis of assignment. The staff member should
personnel actions. Discrimination on participate in the decision and be able
the basis of personal beliefs could to discuss and clarify the assignment.
expose the organization to legal action.
Answer (B) is correct. The supervisor's
Answer (D) is incorrect because expectations should be clear. The staff
personal beliefs alone are not an member should also be involved in
appropriate basis for managerial action. determining how to reach the desired
outcome, thereby increasing both
acceptance and understanding of the
[154] Source: CIA 1196 II-37 assignment.

Answer (A) is incorrect because, if Answer (C) is incorrect because this


members of the group are responsible approach almost guarantees that the
for the decision making, their staff member will feel that (s)he failed
participation in the implementation to perform well.
process will increase the ease with
which the decisions are carried out. Answer (D) is incorrect because the
significant risk is that the staff member
Answer (B) is incorrect because group will not define an outcome and an
decision making adds legitimacy to the approach with which the manager
solution by following democratic agrees.
methods.

Answer (C) is incorrect because a [157] Source: CIA 1196 II-26


Answer (C) is incorrect because expert
Answer (A) is incorrect because an power is possessed by those who can
approach based on pure power is an dispense valued information.
autocratic style of leadership, not a
supportive approach. Answer (D) is incorrect because
legitimate power stems from holding a
Answer (B) is incorrect because the superior position.
custodial model depends on material
rewards for the worker. This model is
predicated on the belief that a happy [160] Source: CIA 1196 II-34
worker is a productive worker.
Answer (A) is incorrect because
Answer (C) is correct. Supportive suspending employees is punishment.
management techniques orient workers
toward performance rather than Answer (B) is correct. Positive
obedience or happiness. The leader reinforcement is a behavior
should have positive feelings for his/her modification technique that provides
employees and should attempt to rewards for certain responses. It
encourage participation and focuses on desirable rather than
involvement. This approach is effective undesirable behavior. The practice of
when used with employees who are praising employees when the detected
motivated to work, improve themselves error rate in their work stays below a
and their abilities, and accomplish predefined level demonstrates positive
goals. reinforcement.

Answer (D) is incorrect because the Answer (C) is incorrect because


manager's beliefs are not sufficient. The eliminating time budgets is extinction,
workers must also believe in the which is the elimination of
system. reinforcement that is maintaining a
behavior.

[158] Source: CIA 1196 II-27 Answer (D) is incorrect because not
requiring employees to work overtime
Answer (A) is incorrect because the is negative reinforcement, which is the
participative approach assumes that elimination of something unpleasant
workers are positively motivated. when a desired behavior occurs.

Answer (B) is incorrect because the


presence of dissatisfiers is not [161] Source: CIA 0596 II-22
consistent with the participative
approach. Answer (A) is incorrect because
optimizing can usually discover more
Answer (C) is correct. For a benefits to divide.
participative management approach to
succeed, the parties must have sufficient Answer (B) is correct. Optimizing or
time, the issues must be relevant to problem solving entails addressing the
employees' interests, employees must source of conflict and finding
have the abilities (training and alternative strategies that benefit all
communication skills) to participate, parties. It promotes cooperative,
and the company culture should support positive attitudes that transfer to other
participation. Accordingly, a limitation organizational behaviors. Hence,
of the participative approach is that it is optimizing may be worth the
unlikely that all employees are willing expenditure of more resources than
to participate in decision making. other strategies because it improves the
future relationship of the parties.
Answer (D) is incorrect because such
conflicts arise when the needs of Answer (C) is incorrect because, to
individuals are not integrated with the optimize, people need to think outside
needs of the organization. of established habits to find new
benefits to divide.

[159] Source: CIA 1196 II-32 Answer (D) is incorrect because


optimizing takes more time and energy
Answer (A) is incorrect because than other conflict resolution strategies.
coercive power is rooted in fear or
threat of punishment.
[162] Source: CIA 1196 II-19
Answer (B) is correct. An individual
has referent power when (s)he Answer (A) is correct. Diffusion
possesses attributes with which others temporarily leaves the conflict
identify or to which they are highly unresolved. Smoothing (downplaying
attracted. Thus, referent power induces differences and emphasizing common
others to comply with an individual's interests) and compromise (requiring
wishes based on that individual's each party to make concessions) are
charisma, not level of authority and diffusion approaches to conflict
expertise. management. The disadvantage is that
the underlying problems remain
unresolved while the less controversial should be the criteria used.
issues are being addressed first.
Answer (C) is incorrect because faculty
Answer (B) is incorrect because members have no control over cost,
differences are downplayed while using capital requirements, or revenue
a diffusion approach. produced.

Answer (C) is incorrect because Answer (D) is incorrect because faculty


diffusion addresses issues of conflict. members have no control over cost,
capital requirements, or revenue
Answer (D) is incorrect because produced.
directly addressing the conflict is a
confrontational approach.
[166] Source: CIA 1196 II-30

[163] Source: CIA 1196 II-33 Answer (A) is incorrect because


evaluating outcomes is desirable when
Answer (A) is correct. The conflict the means are less important than the
management technique that involves ends.
face-to-face meetings is problem
solving. Problem solving is a means of Answer (B) is incorrect because use of
confronting the conflict and removing multiple evaluators increases the
its causes. The emphasis is on facts and probability of obtaining an accurate
solutions, not personalities and appraisal.
assignment of blame.
Answer (C) is incorrect because use of
Answer (B) is incorrect because multiple criteria allows for a more
expansion of resources addresses complete evaluation of quality.
conflicts that arise from scarcity.
Answer (D) is correct. Traits constitute
Answer (C) is incorrect because the one of the weakest sets of criteria for
manager is not using formal authority. evaluating people. Many traits have
little or no relation to performance.
Answer (D) is incorrect because the Furthermore, the evaluation of a trait is
manager is not using behavioral a matter of judgment, and different
techniques to change attitudes and evaluators may have different views of
behavior. such traits as reliability, attitude, and
intelligence.

[164] Source: CIA 0596 II-27


[167] Source: Publisher
Answer (A) is correct. Being
considered a marginal performer may Answer (A) is incorrect because the
cause the supervisor to feel insecure, a change in the level of production
factor that may distort his/her relative to the prior period does not
perception of the worker's performance. necessarily result in a difference in the
That the supervisor becomes more net income reported under variable and
critical as the worker accomplishes absorption costing.
more suggests that the supervisor is
threatened by the worker's superior Answer (B) is correct. When
performance. production exceeds sales, net income
reported under variable costing is
Answer (B) is incorrect because the lower than net income reported under
facts do not suggest that the worker's absorption costing. Under variable
attitude or his behavior is the cause of costing, total fixed manufacturing costs
the misperception. are expensed in the current period.
Under absorption costing, fixed factory
Answer (C) is incorrect because the overhead is inventoried. Thus, when
facts say nothing about the work production exceeds sales, absorption
environment. costing defers some fixed factory
overhead to the following period.
Answer (D) is incorrect because the
facts say nothing about job standards. Answer (C) is incorrect because, when
production equals sales, net income is
the same under both absorption and
[165] Source: CIA 1196 II-28 variable costing.

Answer (A) is incorrect because faculty Answer (D) is incorrect because, when
members have no control over cost. sales exceed production, net income is
lower under absorption costing. Some
Answer (B) is correct. The fixed costs from the prior period are
performance of professional employees, included in beginning inventory.
such as a college faculty, should be
measured using criteria over which they
have some control. In the case of a [168] Source: Publisher
college faculty, the quantity and quality
of output and the timeliness of output Answer (A) is incorrect because
income will be lower under variable
costing when production exceeds sales. Answer (B) is incorrect because ending
inventory is $4,000 under absorption
Answer (B) is incorrect because costing.
income will be higher under variable
costing. Answer (C) is correct. Under
absorption costing, the unit cost of
Answer (C) is incorrect because ending inventory is $40 [$10,000 raw
finished goods inventory will be higher materials + $20,000 direct labor +
under absorption costing. $20,000 variable factory overhead +
$30,000 fixed factory overhead) ÷
Answer (D) is correct. Absorption 2,000 units produced]. Given no
costing always results in a higher beginning inventory, ending inventory is
finished goods inventory because it 100 units (2,000 units produced - 1,900
inventories fixed manufacturing units sold). Hence, ending inventory is
overhead. Under variable costing, $4,000 (100 units x $40).
inventory valuation is lower because
those costs are expensed as incurred. Answer (D) is incorrect because ending
inventory is $4,000 under absorption
costing.
[169] Source: Publisher

Answer (A) is correct. Under variable [172] Source: Publisher


costing, operating income always
moves in the same direction as sales Answer (A) is correct. The difference
volume. The reason is that all costs in profits between the two costing
included in cost of sales are variable methods is due to the treatment of fixed
and therefore are directly related to factory overhead. Under absorption
sales volume. Furthermore, variations costing, $40 per unit in ending inventory
in production levels and inventories do is recognized on the balance sheet,
not affect profits and losses because no whereas only $25 is recognized under
fixed costs are capitalized. variable costing. Hence, the $15
difference multiplied by 100 units
Answer (B) is incorrect because sales remaining in ending inventory causes
volume is directly related to profit absorption costing profits to exceed
under variable costing. variable costing profits by $1,500.

Answer (C) is incorrect because sales Answer (B) is incorrect because $4,000
volume is directly related to profit is ending inventory under absorption
under variable costing. costing.

Answer (D) is incorrect because sales Answer (C) is incorrect because


volume is directly related to profit $30,000 is the total fixed factory
under variable costing. overhead for the period.

Answer (D) is incorrect because


[170] Source: Publisher absorption costing profits will equal
variable costing profits only when sales
Answer (A) is incorrect because ending equal production.
inventory is $2,500 under variable
costing.
[173] Source: Publisher
Answer (B) is correct. Under variable
costing, the unit cost of ending inventory Answer (A) is incorrect because
is $25 [($10,000 raw materials + variable costing profits will exceed
$20,000 direct labor + $20,000 absorption costing profits when
variable factory overhead) ÷ 2,000 inventory quantities decrease during a
units produced]. Given no beginning period.
inventory, ending inventory is 100 units
(2,000 units produced - 1,900 units Answer (B) is correct. Inventory
sold). Hence, ending inventory is quantity will increase when production
$2,500 (100 units x $25). exceeds sales. In that case, profit is
higher under absorption costing than
Answer (C) is incorrect because ending variable costing because some fixed
inventory is $2,500 under variable factory overhead is inventoried.
costing.
Answer (C) is incorrect because the
Answer (D) is incorrect because ending two methods report equal profits when
inventory is $2,500 under variable inventory remains the same during the
costing. period.

Answer (D) is incorrect because


[171] Source: Publisher variable costing reports a lower
inventory value than absorption costing.
Answer (A) is incorrect because ending
inventory is $4,000 under absorption
costing. [174] Source: Publisher
method used.
Answer (A) is incorrect because, under
variable costing, all product costs are Answer (B) is incorrect because, over
variable. the life of a company, total profits will
be the same regardless of the costing
Answer (B) is correct. Proponents of method used.
variable costing maintain that fixed
overhead costs are more closely Answer (C) is correct. Over the entire
correlated to capacity to produce than life of a company, all costs will
to the production of individual units. eventually be expensed. Thus, long-term
Thus, allocation of fixed production total profit will be the same under
costs is arbitrary. variable and absorption costing.

Answer (C) is incorrect because Answer (D) is incorrect because, over


production costs (e.g., direct materials, the life of a company, total profits will
direct labor, and variable factory be the same regardless of the costing
overhead) are deemed to be method used.
controllable.

Answer (D) is incorrect because period [178] Source: Publisher


costs are best attributable to time
intervals but nevertheless are necessary Answer (A) is incorrect because
to production. variable manufacturing overhead costs
are product costs under variable
costing.
[175] Source: Publisher
Answer (B) is correct. Under variable
Answer (A) is incorrect because costing, fixed manufacturing overhead
income is not always greater using the
variable costing method. costs are expensed in the period that
they are incurred. However, all
Answer (B) is correct. Under variable production costs are
absorption costing, managers can inventoriable under variable costing.
manipulate income simply by changing
the amount of units produced. Preparing Answer (C) is incorrect because, under
statements under variable costing variable costing, selling and
allows upper-level management to administrative costs and fixed
better evaluate the activities of manufacturing overhead are period
production managers because some costs.
fixed costs are not hidden in inventory.
Answer (D) is incorrect because
Answer (C) is incorrect because the variable costing inventories direct
method of costing is not useful in materials, direct labor, and variable
evaluating sales personnel. They do not manufacturing overhead.
control production costs.

Answer (D) is incorrect because [179] Source: Publisher


variable costing is not permitted to be
used for income tax purposes. Answer (A) is incorrect because the
effect on absorption costing profits
cannot be determined without more
[176] Source: Publisher data.

Answer (A) is incorrect because all Answer (B) is correct. Under variable
variable manufacturing costs are costing, profit always moves in the
product costs under variable costing. same direction as sales volume.

Answer (B) is incorrect because all


variable manufacturing costs are Answer (C) is incorrect because the
product costs under variable costing. effect on absorption costing profits
cannot be determined without more
Answer (C) is incorrect because all data.
variable manufacturing costs are
product costs under variable costing. Answer (D) is incorrect because
operating profits moves in the same
Answer (D) is correct. Under variable direction as sales volume under
costing, variable costs of production variable costing.
(direct materials, direct labor, and
variable manufacturing overhead) are
treated as inventoriable costs. [180] Source: Publisher

Answer (A) is incorrect because the


[177] Source: Publisher total amount of fixed costs incurred is
reported in the income statement. No
Answer (A) is incorrect because, over fixed costs are absorbed by inventory
the life of a company, total profits will and expensed in a period other than
be the same regardless of the costing when they are incurred.
variance is $3,400 ($166,400 -
Answer (B) is incorrect because profits $163,000). It is favorable since the
and sales move in the same direction actual cost is less than that budgeted.
under variable costing.

Answer (C) is incorrect because the [184] Source: Publisher


total amount of fixed costs incurred is
reported in the income statement. No Answer (A) is incorrect because the
fixed costs are absorbed by inventory variance is unfavorable.
and expensed in a period other than
when they are incurred. Answer (B) is incorrect because the
variance is unfavorable.
Answer (D) is correct. Variable costing
emphasizes total costs by reporting the Answer (C) is incorrect because a
total amount of fixed costs separately in flexible budget fixed cost should not
the income statement. increase when production increases.

Answer (D) is correct. The budgeted


[181] Source: Publisher depreciation expense should be
$100,000 at all levels of production.
Answer (A) is correct. One of the Thus, the variance will be $3,000
advantages of variable costing is the unfavorable ($100,000 budgeted -
emphasis on marginal profit. This $103,000 actual).
emphasis serves as a justification for
changes in production levels. It is also
helpful in evaluating the profitability of [185] Source: CMA Samp Q3-12
segments of a business.
Answer (A) is incorrect because the
Answer (B) is incorrect because it is a sales manager does not have
disadvantage of direct costing. operational control of manufacturing
processes.
Answer (C) is incorrect because it is a
disadvantage of direct costing. Answer (B) is correct. The production
manager is most likely to have influence
Answer (D) is incorrect because it is a over and information about
disadvantage of direct costing. manufacturing quality. This manager
may not control all facets of the
process, for example, the quality of
[182] Source: Publisher machinery or materials, but (s)he has
the knowledge to explain variances
Answer (A) is incorrect because the between actual and expected
contribution margin is the difference performance. (S)he may also be
between sales and total variable costs. motivated to exert pressure on those
who do control success factors.
Answer (B) is correct. The contribution
margin is the difference between sales Answer (C) is incorrect because the
and total variable costs. It indicates division president does not have
how much sales contribute toward operational control of manufacturing
recovering fixed costs and providing a processes.
profit.
Answer (D) is incorrect because the
Answer (C) is incorrect because the receiving and inspection manager does
contribution margin is the difference not have operational control of
between sales and total variable costs. manufacturing processes.

Answer (D) is incorrect because the


contribution margin is the difference [186] Source: Publisher
between sales and total variable costs.
Answer (A) is incorrect because the
board would be consulted initially only
[183] Source: Publisher if the immediate superior is the chief
executive officer and that person is
Answer (A) is incorrect because the involved in the ethical conflict.
total variable delivery expense will
increase as production increases. Answer (B) is correct. The Standards
of Ethical Conduct for Practitioners of
Answer (B) is incorrect because the Management Accounting and Financial
variance is favorable. Management state that the financial
manager/management accountant should
Answer (C) is incorrect because the first discuss an ethical problem with
total variable delivery expense will his/her immediate superior. If the
increase as production increases. superior is involved, the problem
should be taken initially to the next
Answer (D) is correct. The variable higher managerial level.
delivery expense should total $166,400
given sales of 52,000 units ($160,000 ÷ Answer (C) is incorrect because unless
50,000 units = $3.20 per unit). Thus, the "legally prescribed, communication of
such problems to authorities or responsible behavior clearly has
individuals not employed or engaged by immediate costs to the entity, for
the organization is not considered example, the expenses incurred in
appropriate." affirmative action programs, pollution
control, and improvements in worker
Answer (D) is incorrect because safety. When one firm incurs such costs
resignation is a last resort. and its competitor does not, the other
may be able to sell its products or
services more cheaply and increase its
[187] Source: Publisher market share at the expense of the
socially responsible firm. The rebuttal
Answer (A) is incorrect because argument is that in the long run the
"practitioners of management socially responsible company may
accounting and financial management maximize profits by creating goodwill
have an obligation to the public, their and avoiding or anticipating
profession, the organization they serve, governmental regulation.
and themselves, to maintain the highest
standards of ethical conduct."
[190] Source: Publisher
Answer (B) is incorrect because the
audit committee would be consulted Answer (A) is incorrect because this
first only if it were the next higher course of action would be appropriate
managerial level. only for the chief executive officer or
for his/her immediate subordinate when
Answer (C) is correct. To resolve an the CEO is involved in the conflict.
ethical problem, the financial
manager/management accountant's first Answer (B) is incorrect because the
step is usually to consult his/her proper action would be to present the
immediate superior. If that individual is matter to the next higher managerial
involved, the matter should be taken to level.
the next higher level of management.
Answer (C) is incorrect because such
Answer (D) is incorrect because if the action is inappropriate unless legally
superior is involved, the next higher prescribed.
managerial level should be consulted
first. Answer (D) is correct. In these
circumstances, the problem should be
discussed with the immediate superior
[188] Source: Publisher unless (s)he is involved. In that case,
initial presentation should be to the next
Answer (A) is incorrect because a higher managerial level. If the problem
perfectly competitive market was is not satisfactorily resolved after
envisioned by classical economics. initial presentation, the question should
be submitted to the next higher level.
Answer (B) is incorrect because the
concept embraces the public or societal
interest. [191] Source: Publisher

Answer (C) is correct. The concept of Answer (A) is incorrect because it


corporate social responsibility involves applies to external auditors. The IMA
more than serving the interests of the Code of Ethics does not expressly use
organization and its shareholders. such language.
Rather, it is an extension of
responsibility to embrace service to the Answer (B) is correct. The preamble to
public interest in such matters as the IMA Code of Ethics states,
environmental protection, employee "Practitioners of management
safety, civil rights, and community accounting and financial management
involvement. have an obligation to the public, their
profession, the organizations they serve,
Answer (D) is incorrect because the and themselves, to maintain the highest
concept embraces the public or societal standards of ethical conduct. In
interest. recognition of this obligation, the
Institute of Management Accountants
has promulgated the following
[189] Source: Publisher standards of ethical conduct for
practitioners of management accounting
Answer (A) is incorrect because such and financial management. Adherence
behavior may prevent governmental to these standards, both domestically
action. and internationally, is integral to
achieving the Objectives of
Answer (B) is incorrect because each is Management Accounting. Practitioners
an argument for such behavior. of management accounting and financial
management shall not commit acts
Answer (C) is incorrect because each is contrary to these standards nor shall
an argument for such behavior. they condone the commission of such
acts by others within their
Answer (D) is correct. Socially organizations."
not employed or engaged by the
Answer (C) is incorrect because it organization is not considered
applies to external auditors. The IMA appropriate."
Code of Ethics does not expressly use
such language. Answer (B) is correct. According to the
IMA Code of Ethics, financial
Answer (D) is incorrect because it managers/management accountants are
applies to external auditors. The IMA responsible for observing the standard
Code of Ethics does not expressly use of confidentiality. Thus, the financial
such language. manager/management accountant should
"refrain from disclosing confidential
information acquired in the course of
[192] Source: Publisher his/her work except when authorized,
unless legally obligated to do so."
Answer (A) is incorrect because it
states an aspect of the competence Answer (C) is incorrect because the
requirement. financial manager/management
accountant should "inform subordinates
Answer (B) is correct. According to the as appropriate regarding the
IMA Code of Ethics, financial confidentiality of information acquired
managers/management accountants must in the course of their work and monitor
"avoid actual or apparent conflicts of their activities to assure the
interest and advise all appropriate maintenance of that confidentiality."
parties of any potential conflict."
Answer (D) is incorrect because the
Answer (C) is incorrect because it financial manager/management
states an aspect of the confidentiality accountant is required to "refrain from
requirement. using or appearing to use confidential
information acquired in the course of
Answer (D) is incorrect because it his/her work for unethical or illegal
states an aspect of the competence advantage either personally or through
requirement. third parties."

[193] Source: Publisher

Answer (A) is incorrect because, in this


situation, the chief executive officer is
the next higher managerial level.

Answer (B) is incorrect because the


immediate superior has promised or
taken action toward satisfactory
resolution.

Answer (C) is incorrect because the


immediate superior has promised or
taken action toward satisfactory
resolution.

Answer (D) is correct. According to the


IMA Code of Ethics, the financial
manager/management accountant should
"discuss such problems with the
immediate superior except when it
appears that the superior is involved, in
which case the problem should be
presented initially to the next higher
managerial level. If satisfactory
resolution cannot be achieved when the
problem is initially presented, submit
the issues to the next higher managerial
level. If the immediate superior is the
chief executive officer, or equivalent,
the acceptable reviewing authority may
be a group such as the audit committee,
executive committee, board of
directors, board of trustees, or owners."

[194] Source: Publisher

Answer (A) is incorrect because the


IMA Code of Ethics states that "except
where legally prescribed,
communication of such [ethical conflict]
problems to authorities or individuals

You might also like